Silvestri1601-1700

  • Uploaded by: Linda Kuglarz
  • 0
  • 0
  • October 2019
  • PDF

This document was uploaded by user and they confirmed that they have the permission to share it. If you are author or own the copyright of this book, please report to us by using this DMCA report form. Report DMCA


Overview

Download & View Silvestri1601-1700 as PDF for free.

More details

  • Words: 25,731
  • Pages: 60
1

PN~CD~Questions~1601-1700 -

-

Comprehensive Review CD Questions 1601-1700

{ question: 1624, 1631 (Note to developer: the student will need to use a drag and drop feature to answer Question 1631 and list the nursing actions in order of priority. The correct order of action is provided in the answer.); formula: 1624}

1601. A client returns from the recovery room following an abdominal surgical procedure. Following the arrival of the client to the nursing unit, the initial nursing action is to assess the: 1. Abdominal dressing 2. Urinary output in the Foley bag 3. Intravenous (IV) solution for accurate flow rate 4. Vital signs Answer: 4 Rationale: The initial nursing action is to assess the client’s vital signs. The vital signs will provide information regarding airway, breathing, and the circulatory status of the client. Additionally, this data provides a baseline for further assessments. The abdominal dressing, IV, and urine output are also components of the assessment, and these assessments would follow the assessment of the vital signs. Test-Taking Strategy: Use the principles of prioritization when answering this question. Use the ABCs—airway, breathing, and circulation. Vital signs provide data regarding airway, breathing, and circulation. Options 1, 2, and 3 are all nursing actions that should be performed after vital signs. Review care to the postoperative client if you had difficulty with this question. Level of Cognitive Ability: Application Client Needs: Physiological Integrity Integrated Process: Nursing Process/Implementation Content Area: Delegating/Prioritizing References: Christensen, B., & Kockrow, E. (2003). Adult health nursing (4th ed.). St. Louis: Mosby, p. 50. Perry, A., & Potter, P. (2004). Clinical nursing skills & techniques (5th ed.). St. Louis: Mosby, p. 1631. 1602. A nurse is changing the abdominal dressing on a client following a suprapubic prostatectomy. A Penrose drain is in place in the abdominal wound. Which nursing action would be appropriate during the dressing change? 1. Wearing clean gloves during the procedure

PN~CD~Questions~1601-1700 -

2 -

2. Advancing the drain by ¼ inch 3. Checking the wound site for drainage from the drain 4. Securing the drain by taping it firmly to body Answer: 3 Rationale: Usually the drainage from the wound is pale, red, and watery. Active bleeding will be bright red in color. Aseptic technique must be used when changing the dressing to prevent contamination of the wound, and sterile gloves are worn. The drain should be checked for patency to provide an exit for the fluid and blood to promote healing. The drainage needs to flow freely, and there should be no kinks in the drains. Curling, folding, or taping the drain prevents the flow of the drainage. The tube is not advanced. Test-Taking Strategy: Knowledge of the care of drains is necessary to answer this question. Read each option carefully and visualize each option. Note that option 3 is the only option that is a data collection action, the first step in the nursing process. Review nursing care to the client with a Penrose drain if you had difficulty with this question. Level of Cognitive Ability: Application Client Needs: Physiological Integrity Integrated Process: Nursing Process/Implementation Content Area: Fundamental Skills References: Ignatavicius, D., & Workman, M. (2006). Medical-surgical nursing: Critical thinking for collaborative care (5th ed.). Philadelphia: W. B. Saunders, p. 350. Potter, P., & Perry, A. (2005). Fundamentals of nursing (6th ed.). St. Louis: Mosby, p. 1507. 1603. A nurse is assisting in caring for a client immediately following an abdominal surgical procedure who lost a significant amount of blood during surgery. Which finding would indicate a sign of a potential complication? 1. Absent bowel signs 2. A pulse rate of 90 beats/min 3. A blood pressure of 120/70 mm Hg 4. Increasing restlessness Answer: 4 Rationale: Increasing restlessness noted in a client is a sign that requires continuous and close monitoring because it could be indicative of a potential indication of a complication, such as shock. Absent bowel sounds are a normal occurrence in the immediate postoperative period following abdominal surgery. A blood pressure of 120/70 mm Hg with a pulse of 90 beats/min is a relatively normal sign. Test-Taking Strategy: Note the key words immediately, abdominal, and lost a significant amount of blood. Eliminate options 1, 2, and 3 because these are normal expected findings. If you had difficulty with this question, review the normal expected postoperative findings and the signs and symptoms of postoperative complications. Level of Cognitive Ability: Analysis Client Needs: Physiological Integrity Integrated Process: Nursing Process/Data Collection Content Area: Fundamental Skills References: Harkreader, H., & Hogan, M.A. (2004). Fundamentals of nursing: Caring

PN~CD~Questions~1601-1700 -

3 -

and clinical judgment (2nd ed.). Philadelphia: W.B. Saunders, p. 906. Ignatavicius, D., & Workman, M. (2006). Medical-surgical nursing: Critical thinking for collaborative care (5th ed.). Philadelphia: W.B. Saunders, p. 346. Perry, A., & Potter, P. (2004). Clinical nursing skills & techniques (5th ed.). St. Louis: Mosby, p. 982. Potter, P., & Perry, A. (2005). Fundamentals of nursing (6th ed.). St. Louis: Mosby, p. 1642. 1604. A nurse is changing the abdominal dressing on a postoperative client following abdominal surgery. The nurse notes that the incision line is separated and notes the appearance of underlying tissue. Wound dehiscence is suspected. Which of the following is the appropriate initial nursing action? 1. Ask the client to cough to verify the presence of dehiscence 2. Apply a sterile dressing soaked with sterile normal saline to the wound 3. Leave the incision open to the air 4. Apply a dry sterile dressing to the wound Answer: 2 Rationale: Wound dehiscence is the separation of wound edges at the suture line. Signs and symptoms include increased drainage and the appearance of underlying tissue. It usually occurs 6 to 8 days after surgery. The client should be instructed to remain quiet and to avoid coughing or straining. The client should be positioned to prevent further stress on the wound. Sterile dressings soaked with sterile normal saline should be used to cover the wound. The physician needs to be notified. Test-Taking Strategy: Use the process of elimination. Eliminate option 1 because coughing will disrupt the exposed underlying tissue and organs. Eliminate option 3 because this action would expose the open wound and underlying tissue to infection. Eliminate option 4 next. A dry dressing will irritate the exposed body tissue. Review emergency care when dehiscence or evisceration occurs, if you had difficulty with this question. Level of Cognitive Ability: Application Client Needs: Physiological Integrity Integrated Process: Nursing Process/Implementation Content Area: Fundamental Skills References: Harkreader, H., & Hogan, M.A. (2004). Fundamentals of nursing: Caring and clinical judgment (2nd ed.). Philadelphia: W.B. Saunders, pp. 621-622. Potter, P., & Perry, A. (2005). Fundamentals of nursing (6th ed.). St. Louis: Mosby, pp. 1493-1494. 1605. A nurse is preparing a client for surgery. Which of the following would be a component of the plan of care? 1. Review the results of the preoperative laboratory studies 2. Report any increases in blood pressure on the day of surgery 3. Verify that the client has remained NPO for 24 hours before surgery 4. Instruct the client to avoid oral hygiene on the morning of surgery Answer: 1 Rationale: The nurse needs to review the results of the preoperative laboratory studies

PN~CD~Questions~1601-1700 -

4 -

and the physician is notified if any abnormal results are present. Some increase in both blood pressure and pulse is common because of client anxiety regarding surgery. The client usually has a restriction of food and fluids for 8 hours before surgery instead of 24 hours. Oral hygiene is allowed, but the client should not swallow any water. Test-Taking Strategy: Read the options carefully and use the process of elimination to answer the question. Recalling that surgery can produce anxiety in the client will assist in eliminating option 2. Option 4 can be eliminated because there is no reason to avoid oral hygiene as long as the client does not swallow any water. Careful reading of option 3 will assist in eliminating this option and direct you to option 1. Review general preoperative care if you had difficulty with this question. Level of Cognitive Ability: Application Client Needs: Physiological Integrity Integrated Process: Nursing Process/Planning Content Area: Fundamental Skills References: Perry, A., & Potter, P. (2004). Clinical nursing skills & techniques (5th ed.). St. Louis: Mosby, p. 964. Potter, P., & Perry, A. (2005). Fundamentals of nursing (6th ed.). St. Louis: Mosby, p. 1606. 1606. A client with arthritis is scheduled for a surgical knee joint replacement. The client will be admitted to the hospital on the day of the surgical procedure, and the nurse is reinforcing instructions to the client regarding preparation for the surgical procedure. Which statement by the client indicates an understanding of the preoperative instructions? 1. “I cannot drink or eat anything after midnight on the night before surgery.” 2. “My last dose of prescribed acetylsalicylic acid (aspirin) should be taken the evening before surgery.” 3. “I need to stop taking my prescribed prednisone (Deltasone) 48 hours before the scheduled surgery.” 4. “I need to discontinue my prescribed knee exercises at least 1 week before surgery.” Answer: 1 Rationale: Antiplatelet medications, such as acetylsalicylic acid, alter normal clotting factors and increase the risk of hemorrhage. Acetylsalicylic acid has properties that can alter the clotting mechanism and should be discontinued at least 48 hours before surgery. Prednisone, a corticosteroid, should not be abruptly discontinued. In fact it may be necessary to provide additional doses of the corticosteroid before situations that are stressful, such as surgery. There is no reason to discontinue prescribed exercises, and discontinuing exercises in this client may be harmful. The client should be instructed to maintain an NPO status in preparation for surgery. Test-Taking Strategy: Use the process of elimination and eliminate option 4 first because discontinuing exercises can be harmful to the client. Knowledge regarding the medications that affect the surgical client will assist in eliminating options 2 and 3. General principles related to preparing a client for surgery will direct you to option 1. If you had difficulty with this question, review medications that affect the client preparing for surgery and the specific preoperative care to the client with arthritis. Level of Cognitive Ability: Analysis

PN~CD~Questions~1601-1700 -

5 -

Client Needs: Physiological Integrity Integrated Process: Nursing Process/Evaluation Content Area: Fundamental Skills References: Christensen, B., & Kockrow, E. (2003). Adult health nursing (4th ed.). St. Louis: Mosby, p. 25. Perry, A., & Potter, P. (2004). Clinical nursing skills & techniques (5th ed.). St. Louis: Mosby, p. 966. 1607. A nurse is reviewing the arterial blood gas results of the client. Blood gas results indicate a pH of 7.30 and a PC02 of 50 mm Hg, and the nurse has determined that the client is experiencing respiratory acidosis. Which of the following additional laboratory values would the nurse expect to note in this client? 1. Sodium 145 mEq/L 2. Potassium 5.4 mEq/L 3. Magnesium 2.0 mEq/L 4. Phosphorus 2.3 mEq/L Answer: 2 Rationale: Clinical manifestations of respiratory acidosis include dyspnea, disorientation or coma, dysrhythmias, hyperkalemia, and hypoxemia. The sodium, magnesium, and phosphorus would remain within normal range. Test-Taking Strategy: Knowledge regarding the clinical manifestations of respiratory acidosis along with normal laboratory values will assist you in answering the question. By the process of elimination, you can then determine that the only abnormal laboratory value is the potassium level, option 2. Review the clinical manifestations associated with respiratory acidosis and the normal potassium level if you had difficulty with this question. Level of Cognitive Ability: Analysis Client Needs: Physiological Integrity Integrated Process: Nursing Process/Data Collection Content Area: Fundamental Skills References: Black, J., & Hawks, J. (2005). Medical-surgical nursing: Clinical management for positive outcomes (7th ed.). Philadelphia: W.B. Saunders, pp. 254, 1581. Chernecky, C., & Berger, B. (2004). Laboratory tests and diagnostic procedures (4th ed.). Philadelphia: W.B. Saunders, pp. 887-888. 1608. A nurse is changing an abdominal dressing on a client with an open incision and notes the presence of sanguineous drainage. Which nursing action would be appropriate? 1. Clean the wound and cover with a dry sterile dressing 2. Notify the registered nurse (RN) 3. Cover the wound and reassess in 1 hour 4. Leave the wound open to air to assist in drying Answer: 2 Rationale: Sanguineous drainage is bright red and indicates active bleeding. If active bleeding is present, the RN should be notified. Covering the wound and reassessing in 1 hour will delay needed intervention. Leaving a wound open to air can lead to infection.

PN~CD~Questions~1601-1700 -

6 -

Test-Taking Strategy: Use knowledge of the various types of wound drainage to answer this question. Knowing that sanguineous drainage is bright red and indicates active bleeding will direct you to option 2. If you are unfamiliar with the types of wound drainage and the associated interventions, review this content. Level of Cognitive Ability: Application Client Needs: Physiological Integrity Integrated Process: Nursing Process/Implementation Content Area: Fundamental Skills Reference: Perry, A., & Potter, P. (2004). Clinical nursing skills & techniques (5th ed.). St. Louis: Mosby, pp. 1021, 1030. 1609. A nurse is a certified basic life support (BLS) instructor and is conducting a BLS course for health care providers. The nurse determines that a student in the group correctly performs cardiopulmonary resuscitation (CPR) on an infant when the nurse observes that the rate of chest compression delivered to the infant manikin is: 1. Fifty times per minute 2. Eighty times per minute 3. One hundred times per minute 4. One hundred and sixty times per minute Answer: 3 Rationale: In an infant, the rate of chest compressions is at least 100/min. Options 1, 2, and 4 are incorrect. Test-Taking Strategy: Consider the normal heart rate of an infant to answer this question. You can easily eliminate option 1 and 2 because of the low rates identified in the options. Eliminate option 4 because this rate would be much too rapid for an infant. If you had difficulty with this question, review BLS for an infant. Level of Cognitive Ability: Comprehension Client Needs: Physiological Integrity Integrated Process: Teaching/Learning Content Area: Fundamental Skills Reference: Perry, A., & Potter, P. (2004), Clinical nursing skills & techniques (5th ed.). St. Louis: Mosby, p. 432. 1610. A nurse is assisting in caring for a client with a severe burn who has just received an autograft to the knee area of the right leg. The nurse plans to keep the right leg positioned in which manner? 1. Elevated only when out of bed to the chair 2. Elevated and immobilized 3. Flat at all times 4. Dependent and covered with a blanket Answer: 2 Rationale: Autografts placed over joints or on lower extremities are elevated and immobilized following surgery for 3 to 7 days, depending on the surgeon’s preference. This period of immobilization allows the autograft time to adhere and attach to the wound bed, and the elevation minimizes edema. The other options are incorrect because they could, in varying degrees, increase edema to the surgical area.

7

PN~CD~Questions~1601-1700 -

-

Test-Taking Strategy: Use the process of elimination. Option 4 can be eliminated first because a blanket can easily disrupt a graft. From the remaining options, recall the principles related to gravity and edema to assist in directing you to option 2. Review care to the client following an autograft if you had difficulty with this question. Level of Cognitive Ability: Application Client Needs: Physiological Integrity Integrated Process: Nursing Process/Planning Content Area: Adult Health/Integumentary References: Black, J., & Hawks, J. (2005). Medical-surgical nursing: Clinical management for positive outcomes (7th ed.). Philadelphia: W.B. Saunders, p. 1460. Thompson, J., McFarland, G., Hirsch, J., & Tucker, S. (2002). Mosby’s clinical nursing (5th ed.). St. Louis: Mosby, p. 503. 1611. A nurse is caring for a client with a diagnosis of end-stage renal disease. The client tells the nurse that a lawyer has prepared a living will and will be visiting the client today so that the will can be reviewed. The client also tells the nurse that the lawyer has asked for a witness to sign the will and requests that the nurse act as a witness. The most appropriate nursing response to the client is: 1. “I would be pleased to do that for you.” 2. “I never sign anything, and I need to refuse to do this too.” 3. “A nurse caring for a client cannot serve as a witness to a living will.” 4. “You need to talk to the nursing supervisor.” Answer: 3 Rationale: Living wills address the withdrawal or withholding of life-sustaining interventions that unnaturally prolong life. It identifies the person who will make care decisions if the client is unable to take action. It is witnessed and signed by two persons who are unrelated to the client. Nurses or employees of a facility in which the client is receiving care and beneficiaries of the client must not serve as a witness. Test-Taking Strategy: Knowledge of the procedure related to living wills will assist in answering the question. The question asks for the “most appropriate action.” You can easily eliminate option 2 because of the word “refuse.” Option 1 is clearly incorrect. There is no need to call the supervisor, and it is most appropriate to inform the client of the nurse’s role in this procedure. Review the procedures related to living wills if you had difficulty with this question. Level of Cognitive Ability: Application Client Needs: Safe, Effective Care Environment Integrated Process: Nursing Process/Implementation Content Area: Fundamental Skills References: Harkreader, H., & Hogan, M.A. (2004). Fundamentals of nursing: Caring and clinical judgment (2nd ed.). Philadelphia: W.B. Saunders, p. 36. Potter, P., & Perry, A. (2005), Fundamentals of nursing (6th ed.). St. Louis: Mosby, pp. 409-410. 1612. A client with chronic obstructive pulmonary disease (COPD) asks the nurse for assistance with preparing a living will. The client tells the nurse that she has not discussed the living will with the family and wanted to make some decisions before

PN~CD~Questions~1601-1700 -

8 -

discussing the will with the family. The nurse informs the client that the initial step in preparing this document is to: 1. Consult with the American Lung Association 2. Talk to the family 3. Contact a lawyer 4. Discuss the request with the physician Answer: 4 Rationale: The client should discuss the request for a living will with the physician. The client should also discuss this desire with the family, although in this situation, based on the client’s feelings, talking to the family would be the second step. Wills should be prepared with legal counsel and should identify the executor of the estate, address, distribution and use of property, and specific plans for burial. Although option 1 may be helpful, this contact would not be the initial step. The lawyer would be contacted following discussion with the physician and family. Test-Taking Strategy: Use the process of elimination noting the key words initial step. Remembering that the physician is the primary care provider may assist in directing you to the correct option. In this situation, family discussion would follow discussion with the physician. Contacts addressed in options 1 and 3 would then follow. Review the procedure related to preparing living wills if you had difficulty with this question. Level of Cognitive Ability: Application Client Needs: Safe, Effective Care Environment Integrated Process: Nursing Process/Implementation Content Area: Fundamental Skills References: Elkin, M., Perry, A., & Potter, P. (2004). Nursing interventions and clinical skills (3rd ed.). St. Louis: Mosby, p. 929. Perry, A., & Potter, P. (2004), Clinical nursing skills & techniques (5th ed.). St. Louis: Mosby, pp. 1254-1255. 1613. A client with metastatic bladder cancer is admitted to the hospital for chemotherapy. During data collection, the client tells the nurse that a living will was prepared 2 years ago and asks the nurse if this document is still effective. The appropriate nursing response is which of the following? 1. “Yes it is effective until you make the decision to change it.” 2. “You will have to discuss the issue with your lawyer.” 3. “A living will needs to be reviewed yearly with your physician.” 4. “Living wills are valid for 6 months.” Answer: 3 Rationale: The client should discuss the living will with the physician, and it should be reviewed annually to ensure that it contains the client’s present wishes and desires. Options 1 and 4 are inaccurate. Option 2 is not the most appropriate response. Test-Taking Strategy: Use the process of elimination. Options 1 and 4 are incorrect. Although a lawyer would need to be consulted if the living will needed to be changed, the most appropriate and accurate nursing response would be to inform the client that the living will should be reviewed annually with the physician. Review procedures related to living wills if you had difficulty with this question. Level of Cognitive Ability: Application

9

PN~CD~Questions~1601-1700 -

-

Client Needs: Safe, Effective Care Environment Integrated Process: Nursing Process/Implementation Content Area: Fundamental Skills Reference: Perry, A., & Potter, P. (2004). Clinical nursing skills & techniques (5th ed.). St. Louis: Mosby, p. 1254. 1614. {PLACE FIGURE HERE (Fig. 16)} Perry, A., & Potter, P. (2002). Clinical nursing skills & techniques (5th ed.). St. Louis: Mosby, p. 883. 1614. A nurse is preparing to collect data from a client who has sustained a pelvic fracture following a motor vehicle crash. The nurse reviews the physician’s orders and notes that the physician has prescribed a pelvic (skin) sling. The nurse prepares to place the client in which device? Answer: 3 Rationale: A pelvic sling is a traction device consisting of a hammocklike belt wherein the sling cradles the pelvis in its boundaries. It is used for the treatment of one or more pelvic fractures. Option 1 identifies a cervical halter skin traction. Option 2 identifies a pelvic belt traction. Option 4 identifies Russell traction. Test-Taking Strategy: Use the process of elimination. Note the relationship between the word “sling” in the question and the components of the device in option 3. If you had difficulty with this question, review this type of device used to treat pelvic fractures. Level of Cognitive Ability: Application Client Needs: Physiological Integrity Integrated Process: Nursing Process/Planning Content Area: Adult Health/Musculoskeletal Reference: Perry, A., & Potter, P. (2002). Clinical nursing skills & techniques (5th ed.). St. Louis: Mosby, pp. 881, 883. {PLACE FIGURE HERE (Fig. 16). For Q#1614.} Perry, A., & Potter, P. (2002). Clinical nursing skills & techniques (5th ed.). St. Louis: Mosby, p. 883. 1615. A nurse is reviewing the laboratory results of a client hospitalized with Crohn’s disease. The client has a magnesium level of 1.3 mg/dl. The appropriate nursing intervention is to: 1. Encourage increased intake of phosphate antacids 2. Monitor the client for dysrhythmias 3. Instruct the client to substitute ground beef with tofu 4. Instruct the client to avoid eggs or chicken breast Answer: 2 Rationale: Hypomagnesemia is defined as a plasma magnesium level below 1.5 mEq/L or 1.8 mg/dl. Phosphate use should be limited in the presence of hypomagnesemia because it worsens the state. The client should be monitored for dysrhythmias since the client is predisposed particularly to ventricular dysrhythmias. Tofu is a high-magnesium

PN~CD~Questions~1601-1700 -

10 -

food. Ground beef, eggs, and chicken breast are examples of foods that are low in magnesium. Test-Taking Strategy: To answer this question accurately, it is necessary to know the normal magnesium level and determine the appropriate interventions when the level is low. If you did not know the answer to this question, use the ABCs—airway, breathing, and circulation— to answer the question. This will direct you to option 2. If this question was difficult, review this electrolyte disorder and its treatment. Level of Cognitive Ability: Application Client Needs: Physiological Integrity Integrated Process: Nursing Process/Implementation Content Area: Fundamental Skills Reference: Black, J., & Hawks, J. (2005). Medical-surgical nursing: Clinical management for positive outcomes (7th ed.). Philadelphia: W.B. Saunders, pp. 243, 818. 1616. A nurse is collecting data from a client and is observing the client ambulate with the use of a cane. The nurse would intervene and suggest a physical therapy referral if the nurse observed that: 1. The cane handle was parallel to the greater trochanter of the femur 2. The client holds the cane on the unaffected side 3. The client holds the cane close to the body 4. The client moves the cane and the unaffected side together Answer: 4 Rationale: The client should move the cane and the affected side together. The cane helps to support the affected side as it moves forward. It also helps the client to maintain balance. The client holds the cane on the unaffected side to shift the client’s weight away from the affected side. The client holds the cane close to the body to prevent the client from leaning. The cane’s handle should reach the level of the greater trochanter of the client’s femur with a 25- to 30-degree flexion at the client’s elbow. Test-Taking Strategy: Note the key word intervene. This word indicates a false-response question and that you need to select the incorrect action or measure regarding the use of a cane. Visualize each of the options in determining the correct option. If you had difficulty with this question, review the procedure for use of a cane. Level of Cognitive Ability: Comprehension Client Needs: Physiological Integrity Integrated Process: Teaching/Learning Content Area: Fundamental Skills Reference: Harkreader, H., & Hogan, M.A. (2004). Fundamentals of nursing: Caring and clinical judgment (2nd ed.). Philadelphia: W.B. Saunders, pp. 792-793. 1617. A nurse is caring for a child following a tonsillectomy who has wrist restraints in place. The nurse assists in preparing a plan of care and determines that which nursing intervention would receive highest priority regarding the restraints? 1. Providing range-of-motion exercises to the wrists 2. Removing the restraints periodically 3. Applying lotion to the skin under the restraints

PN~CD~Questions~1601-1700 -

11 -

4. Checking color, sensation, and pulses distal to the restraint Answer: 4 Rationale: Checking color, sensation, and pulses distal to the restraint would identify the complication of neurovascular compromise that is associated with the use of restraints. Of the options presented, this is the priority. All of the other interventions should also be implemented and remember that restraints if used should be removed at least every 2 hours or as specified by the agency policy and procedure. Test-Taking Strategy: Note the key words highest priority. Use the ABCs—airway, breathing, and circulation—to answer the question. Checking color, sensation, and pulses would be the highest priority because it determines circulation. Review procedures related to the use of restraints if you had difficulty with this question. Level of Cognitive Ability: Analysis Client Needs: Physiological Integrity Integrated Process: Nursing Process/Planning Content Area: Fundamental Skills Reference: Harkreader, H., & Hogan, M.A. (2004). Fundamentals of nursing: Caring and clinical judgment (2nd ed.). Philadelphia: W.B. Saunders, p. 512. 1618. A client has had extensive surgery on the gastrointestinal tract and has been started on total parenteral nutrition (TPN). The client tells the nurse, “I think I’m going crazy...I feel like I’m starving and yet that bag is supposed to be feeding me.” The best response of the nurse would be: 1. “Don’t worry. Many others in your situation say the same thing.” 2. “That is unusual. I wonder if the solution is being mixed correctly?” 3. “Maybe you should ask your physician about that; I’ve never heard of that before.” 4. “That is because the empty stomach sends signals to the brain to stimulate hunger.” Answer: 4 Rationale: The stomach does send signals to the brain when it is empty to stimulate hunger. The client should be told that this is normal. Some clients also experience food cravings for the same reason. Options 1 and 3 will block the communication process. Option 2 will produce fear in the client. Test-Taking Strategy: Use the process of elimination and therapeutic communication techniques to answer the question. Begin to answer this question by eliminating option 2 first. This response can frighten the client and lessen trust in the health care team. Next, eliminate options 1 and 3. These statements block communication and do not respond to the client’s concerns. Option 4 is the only response that acknowledges the client’s concern and addresses it. Review therapeutic communication techniques and the physiological principles related to TPN if you had difficulty with this question. Level of Cognitive Ability: Application Client Needs: Psychosocial Integrity Integrated Process: Nursing Process/Implementation Content Area: Fundamental Skills References: Mosby’s medical, nursing, & allied health dictionary (2002) (6th ed.). St. Louis: Mosby, p. 1728. Potter, P., & Perry, A. (2005). Fundamentals of nursing (6th ed.). St. Louis: Mosby, p. 437.

12

PN~CD~Questions~1601-1700 -

-

1619. {PLACE FIGURE HERE (Fig. 17)} Perry, A., & Potter, P. (2002). Clinical nursing skills & techniques (5th ed.). St. Louis: Mosby, p. 152. 1619. A nurse is preparing to perform nasotracheal suctioning on a client. The nurse places the client’s bed in which position to most effectively perform this procedure? Answer: 1 Rationale: The client is placed in the Fowler’s position for nasotracheal suction. This position promotes lung expansion and is also the preferred position for eating and for nasogastric tube insertion. Option 2 identifies the Trendelenburg’s position. This position is used for postural drainage. It also facilitates venous return in clients with poor peripheral circulation. Option 3 identifies reverse Trendelenburg’s position. This position promotes gastric emptying and prevents esophageal reflux. Option 4 identifies a flat position that is used for clients with vertebral injuries and for clients in cervical traction. A flat position may also be used for clients who are hypotensive. Test-Taking Strategy: Use the process of elimination. Focus on the nursing procedure to be performed, nasotracheal suctioning. Recalling that the Fowler’s position promotes lung expansion will direct you to option 1. If you had difficulty with this question, review the procedure for nasotracheal suctioning. Level of Cognitive Ability: Application Client Needs: Physiological Integrity Integrated Process: Nursing Process/Implementation Content Area: Adult Health/Respiratory Reference: Perry, A., & Potter, P. (2002). Clinical nursing skills & techniques (5th ed.). St. Louis: Mosby, p. 152. {PLACE FIGURE HERE (Fig. 17). For Q#1619} Perry, A., & Potter, P. (2002). Clinical nursing skills & techniques (5th ed.). St. Louis: Mosby, p. 152. 1620. A nurse is assisting in caring for a client with severe hyponatremia resulting from hypervolemia. The client is being treated with an intravenous hypertonic saline solution. The nurse determines that the treatment measures are effective when the laboratory results reveal a serum sodium level of: 1. 120 mEq/L 2. 130 mEq/L 3. 140 mEq/L 4. 150 mEq/L Answer: 3 Rationale: Hyponatremia is defined as a serum sodium level of less than 135 mEq/L. When it is caused by hypervolemia, it may be treated with fluid restriction. The low serum sodium value is due to hemodilution. Intravenous hypertonic saline (3%) is reserved for hyponatremia when the serum sodium level is lower than 125 mEq/L. The normal serum sodium level is 135 to 145 mEq/L. Options 1 and 2 identify hyponatremia.

PN~CD~Questions~1601-1700 -

13 -

Option 4 indicates hypernatremia. Test-Taking Strategy: To answer this question accurately, it is necessary to know the normal serum sodium level. This knowledge will direct you to option 3. Review this normal level and the treatment measures for hyponatremia if you had difficulty with this question. Level of Cognitive Ability: Analysis Client Needs: Physiological Integrity Integrated Process: Nursing Process/Evaluation Content Area: Fundamental Skills Reference: Black, J., & Hawks, J. (2005). Medical-surgical nursing: Clinical management for positive outcomes (7th ed.). Philadelphia: W.B. Saunders, pp. 226-228. 1621. A client with a history of seizure disorder is taking phenytoin (Dilantin). The nurse reviews the laboratory results of the phenytoin level and determines that the client has been noncompliant with medication therapy if which of the following laboratory results were noted? 1. 5 mcg/mL 2. 10 mcg/mL 3. 16 mcg/mL 4. 19 mcg/mL Answer: 1 Rationale: The therapeutic range for serum phenytoin (Dilantin) level is 10 to 20 mcg/mL. Option 1 is the only option that identifies a low level, indicating that the client is not compliant with the medication regimen. Options 2, 3, and 4 indicate therapeutic levels. Test-Taking Strategy: To answer this question accurately, specific knowledge is needed about the normal range of results for this laboratory test. Note the key word noncompliant. This will direct you to the option that identifies a low value. Learn this therapeutic level if you had difficulty with this question. Level of Cognitive Ability: Analysis Client Needs: Physiological Integrity Integrated Process: Nursing Process/Evaluation Content Area: Fundamental Skills Reference: Hodgson, B., & Kizior, R. (2005). Saunders nursing drug handbook 2005. Philadelphia: W.B. Saunders, p. 858. 1622. A client who takes theophylline (Theo-Dur) for chronic obstructive pulmonary disease (COPD) is seen in the health care clinic. A theophylline level is drawn, and the nurse determines that the client is compliant with the medication regimen if which of the following laboratory results are reported? 1. 2 mcg/mL 2. 5 mcg/mL 3. 8 mcg/mL 4. 15 mcg/mL Answer: 4

PN~CD~Questions~1601-1700 -

14 -

Rationale: The therapeutic range for the serum theophylline level is 10 to 20 mcg/mL. If the level is below the therapeutic range, the client may be noncompliant with the medication regimen. If the level is within the therapeutic range, the client is most likely compliant with medication therapy. Options 1, 2, and 3 identify low serum levels of theophylline. Test-Taking Strategy: Note that the key word in this question is compliant. Familiarity with the therapeutic level of theophylline is needed to select the correct option. Remember that the therapeutic range for the serum theophylline level is 10 to 20 mcg/mL. Review this therapeutic range if you had difficulty with this question. Level of Cognitive Ability: Analysis Client Needs: Physiological Integrity Integrated Process: Nursing Process/Evaluation Content Area: Fundamental Skills Reference: Hodgson, B., & Kizior, R. (2005). Saunders nursing drug handbook 2005. Philadelphia: W.B. Saunders, p. 53. 1623. A nurse is caring for a client with chronic congestive heart failure who is taking digoxin (Lanoxin) 0.125 mg daily. Before administering the medication, the nurse reviews the serum digoxin level that was drawn earlier in the day. The result is 1.0 ng/mL. Which of the following actions will the nurse take based on this laboratory result? 1. Obtain another serum digoxin level to verify the results 2. Administer the dose of the medication as scheduled 3. Check the client’s last pulse rate 4. Notify the physician Answer: 2 Rationale: The normal therapeutic range for digoxin is 0.5 to 2 ng/mL. A value of 1.0 is within therapeutic range, and the nurse would administer the next dose as scheduled. Options 1 and 4 are unnecessary. An apical pulse must be obtained before administering each dose of digoxin. It is incorrect to administer the digoxin based on the client’s last pulse rate, although a comparison of pulse rates may be appropriate. Test-Taking Strategy: To answer this question correctly, it is necessary to be familiar with the therapeutic serum range for this medication. Knowing that the normal therapeutic range for digoxin is 0.5 to 2 ng/mL will direct you to option 2. If this question was difficult, review the information on this commonly used medication and measurement of its therapeutic serum level. Level of Cognitive Ability: Application Client Needs: Physiological Integrity Integrated Process: Nursing Process/Implementation Content Area: Fundamental Skills Reference: Hodgson, B., & Kizior, R. (2005). Saunders nursing drug handbook 2005. Philadelphia: W.B. Saunders, p. 326. 1624. {PLACE FIGURE . HERE (Fig. 18). For Q#1624} Kee, J., & Marshall, S. (2004). Clinical calculations: With applications to general and specialty areas (5th ed.). Philadelphia: W.B. Saunders, p 129.

PN~CD~Questions~1601-1700 -

15 -

Amoxil Medication Label 1624. A physician has prescribed a liquid oral suspension of amoxicillin (Amoxil). The order reads 0.25 g orally three times daily. The nurse reads the label on the medication bottle and administers how many milliliters to the client to administer one dose? Answer: 5 Rationale: Change grams to milligrams then use the formula for calculating medication doses. Grams to milligrams: 0.25 g = 250 mg Formula: Desired _________ × mL = mL per dose Available 250 mg 1250 _____ × 5 mL = _____ = 5 mL 250 mg 250 Test-Taking Strategy: Read the physician’s order and the medication label noting that it is first necessary to convert grams to milligrams. Next, follow the formula for the calculation of the correct dose. Recheck your work using a calculator and make sure that the answer makes sense. If you had difficulty with this question, review medication calculation problems. Level of Cognitive Ability: Application Client Needs: Physiological Integrity Integrated Process: Nursing Process/Implementation Content Area: Fundamental Skills Reference: Kee, J., & Marshall, S. (2004). Clinical calculations: With applications to general and specialty areas (5th ed.). Philadelphia: W.B. Saunders, p. 80. 1625. A nurse is caring for a hospitalized client with a mechanical heart valve who is receiving maintenance therapy of warfarin sodium (Coumadin). The client’s international normalized ratio (INR) is 3.0. The nurse anticipates which of the following orders? 1. Holding the next dose of warfarin sodium 2. Administering the next dose of warfarin sodium 3. Increasing the next dose of warfarin sodium 4. Adding a dose of heparin Answer: 2 Rationale: An INR of 2.0 to 3.0 is appropriate for most clients. An INR of 3.0 to 4.5 is recommended for clients with mechanical heart valves. If the INR is below the recommended range, the warfarin sodium dose would be increased. If the INR is above the recommended range, the warfarin sodium dose would be decreased. Since the value identified in this question is within the therapeutic range, the nurse would administer the next dose of warfarin. Test-Taking Strategy: To answer this question accurately, it is necessary to be familiar with the normal INR and the therapeutic level necessary following institution of warfarin sodium therapy. Remember an INR of 3.0 to 4.5 is recommended for clients with

PN~CD~Questions~1601-1700 -

16 -

mechanical heart valves. If this question was difficult, review this content area. Level of Cognitive Ability: Analysis Client Needs: Physiological Integrity Integrated Process: Nursing Process/Planning Content Area: Fundamental Skills References: Hodgson, B., & Kizior, R. (2005). Saunders nursing drug handbook 2005. Philadelphia: W.B. Saunders, pp. 1122-1123. McKenry, L., & Salerno, E. (2001). Mosby’s pharmacology in nursing (21st ed.). St. Louis: Mosby, p.631. 1626. A nurse is caring for a client who is receiving intramuscular antibiotics. The nurse enters the client’s room to administer the prescribed antibiotic, and the client tells the nurse that the medication burns and that he does not want the medication to be given. The nurse tells the client that the medication is necessary and administers the medication. Which of the following can the client legally charge the nurse with as a result of the nursing action? 1. Assault 2. Battery 3. Negligence 4. Invasion of privacy Answer: 2 Rationale: An assault occurs when a person puts another person in fear of a harmful or offensive contact. For this intentional tort to be actionable, the victim must be aware of the threat of harmful or offensive contact. Battery is the actual contact with one’s body. Negligence involves actions below the standards of care. Invasion of privacy occurs when the individual’s private affairs are unreasonably intruded into. In this situation, the nurse can be charged with battery because the nurse administers a medication that the client has refused. Test-Taking Strategy: Note that the client refuses the medication, and the nurse administers the medication regardless of the client’s request. This should direct you to option 2. If you had difficulty with this question, review the descriptions associated with the terms in each option. Level of Cognitive Ability: Comprehension Client Needs: Safe, Effective Care Environment Integrated Process: Nursing Process/Implementation Content Area: Fundamental Skills References: Harkreader, H., & Hogan, M.A. (2004). Fundamentals of nursing: Caring and clinical judgment (2nd ed.). Philadelphia: W.B. Saunders, p. 17. Potter, P., & Perry, A. (2005). Fundamentals of nursing (6th ed.). St. Louis: Mosby, p. 413. 1627. A nurse overhears a client ask the physician if the results of a biopsy indicated cancer. The physician tells the client that the results have not returned, when in fact the physician is aware that the results of the biopsy indicated the presence of malignancy. The nurse is upset that the physician has not shared the results with the client and tells another nurse that the physician has lied to the client and that this physician probably lies

PN~CD~Questions~1601-1700 -

17 -

to all of the clients. Which legal tort has the nurse violated by this statement? 1. Libel 2. Slander 3. Assault 4. Negligence Answer: 2 Rationale: Defamation takes place when something untrue is said (slander) or written (libel) about a person resulting in injury to that person’s good name and reputation. An assault occurs when a person puts another person in fear of a harmful or an offensive contact. Negligence involves the actions of professionals that fall below the standard of care for a specific professional group. Although the physician may be aware of the biopsy results, the physician decides when it is best to share such a diagnosis with the client. Test-Taking Strategy: Use the process of elimination. You should easily eliminate options 3 and 4 first. Recalling that slander constitutes verbal defamation will direct you to option 2. If you had difficulty with this question, review the torts identified in each option. Level of Cognitive Ability: Comprehension Client Needs: Safe, Effective Care Environment Integrated Process: Nursing Process/Implementation Content Area: Fundamental Skills Reference: Potter, P., & Perry, A. (2005). Fundamentals of nursing (6th ed.). St. Louis: Mosby, p. 414. 1628. A nurse is preparing to administer medications to an assigned client and notes that the order for furosemide (Lasix) is higher than the recommended dosage. The nurse calls the physician to clarify the order and asks the physician to prescribe a dosage within the recommended range. The physician refuses to change the order and instructs the nurse to administer the dose as prescribed. Which of the following actions would the nurse take? 1. Discontinue the order 2. Administer the dose as prescribed 3. Call the state medical board and report the physician 4. Contact the nursing supervisor Answer: 4 Rationale: If the physician writes an order that requires clarification, it is the nurse’s responsibility to contact the physician for clarification. If there is no resolution regarding the order because the order remains as it was written after talking with the physician or because the physician cannot be located, the nurse should then contact the nurse manager or supervisor for further clarification as to what the next step should be. Under no circumstances should the nurse proceed to carry out the order until clarification is obtained. Option 1 is not within the scope of nursing practice. Option 3 is a premature action. Test-Taking Strategy: Use the process of elimination. Eliminate option 2 first because this is an unsafe action. Eliminate option 1 next because this action is outside the scope of nursing practice. Option 3 is premature and should be eliminated. Additionally the nurse should follow the organizational chain of command and seek assistance from the

PN~CD~Questions~1601-1700 -

18 -

nursing supervisor. Review nursing responsibilities related to physician’s orders if you had difficulty with this question. Level of Cognitive Ability: Application Client Needs: Safe, Effective Care Environment Integrated Process: Nursing Process/Implementation Content Area: Fundamental Skills Reference: Potter, P., & Perry, A. (2005). Fundamentals of nursing (6th ed.). St. Louis: Mosby, p. 419. 1629. A new nurse graduate asks another nurse about the need to obtain professional liability insurance. The most appropriate response by the nurse is: 1. “The hospital insurance covers your actions.” 2. “It is very expensive, and you really don’t need it since the hospital covers you.” 3. “Nurses need to have their own malpractice insurance.” 4. “The lawsuits are filed against physicians and the hospital, so you are safe not to obtain it.” Answer: 3 Rationale: Nurses need their own liability insurance for protection against malpractice law suits. Nurses erroneously assume that they are protected by agencies’ professional liability policies. Usually when a nurse is sued, the employer is also sued for the nurse’s actions or inactions. Even though this is the usual practice, nurses are encouraged to have their own malpractice insurance. Test-Taking Strategy: Note that the issue of the question relates to “obtaining professional liability insurance.” This issue should easily direct you to option 3. Note the similarity between options 1, 2, and 4 in that they refer to not obtaining the malpractice insurance. Review liability related to malpractice insurance if you had difficulty with this question. Level of Cognitive Ability: Application Client Needs: Safe, Effective Care Environment Integrated Process: Nursing Process/Implementation Content Area: Leadership/Management Reference: Potter, P., & Perry, A. (2005). Fundamentals of nursing (6th ed.). St. Louis: Mosby, p. 418. 1630. A nurse witnesses an accident in which the victim was hit by a car. The nurse stops at the scene of the accident and administers safe care to a victim who sustained a compound fracture of the femur. The victim is hospitalized, and later sepsis develops as a result of the fractured femur. The victim files suit against the nurse who provided care at the scene of the accident. Which of the following most accurately describes the nurse’s immunity from this suit? 1. The Good Samaritan law will not protect the nurse 2. The Good Samaritan law will protect the nurse if the care given at the scene was not negligent 3. The Good Samaritan law will not provide immunity from suit if the nurse accepted compensation for the care provided 4. The Good Samaritan law protects lay persons and not professional health care

PN~CD~Questions~1601-1700 -

19 -

providers Answer: 2 Rationale: A Good Samaritan law is passed by a state legislator to encourage nurses and other health care providers to provide care to a person when an accident, emergency, or injury occurs without fear of being sued for the care provided. Called immunity from suit, this protection usually applies only if all of the conditions of the law are met, such as the heath care provider receives no compensation for the care provided, and the care given is not willfully and wantonly negligent. Test-Taking Strategy: Focus on the information in the question and use the process of elimination. Note the key words most accurately. This tells you that one or more than one option may be correct. Options 1 and 4 are similar and incorrect statements and can be eliminated first. Eliminate option 3 next because although this is an accurate statement, there is no data in the question that states that the nurse accepted compensation for the care provided. Review the Good Samaritan law if you had difficulty with this question. Level of Cognitive Ability: Comprehension Client Needs: Safe, Effective Care Environment Integrated Process: Nursing Process/Implementation Content Area: Fundamental Skills References: Harkreader, H., & Hogan, M.A. (2004). Fundamentals of nursing: Caring and clinical judgment (2nd ed.). Philadelphia: W.B. Saunders, p. 26. Potter, P., & Perry, A. (2005). Fundamentals of nursing (6th ed.). St. Louis: Mosby, pp. 411-412. 1631. A nurse working in a long-term care facility responds after hearing someone calling, “Help, the bed is on fire!” Upon entering the room, the nurse finds an older client slapping at flames on the bedspread with a pillow. Both hands have been burned. List in order of priority the actions that the nurse takes. Remove the client from the room Close the door to the room Pull the nearest fire alarm Run to get the nearest fire extinguisher Answer: Remove the client from the room Pull the nearest fire alarm Close the door to the room Run to get the nearest fire extinguisher Rationale: In a fire emergency, the steps to follow use the acronym RACE. The first step is to remove the victim. The other steps are: activate the alarm, contain the fire, then extinguish the fire. This is a universal standard that may be applied to any type of fire emergency. Test-Taking Strategy: Sequence the activities using the RACE acronym. Review fire safety if you had difficulty with this question. Level of Cognitive Ability: Application Client Needs: Safe, Effective Care Environment Integrated Process: Nursing Process/Implementation

20

PN~CD~Questions~1601-1700 -

-

Content Area: Fundamental Skills Reference: Harkreader, H., & Hogan, M.A. (2004). Fundamentals of nursing: Caring and clinical judgment (2nd ed.). Philadelphia: W.B. Saunders, p. 505. 1632. An adult client is brought to the emergency room by ambulance after being hit by a car. The client is unconscious and is in shock. A perforated spleen is suspected, and emergency surgery is required immediately to save the client’s life. There are no family members present, but the nurse found identification on the client. In regard to informed consent for the surgical procedure, the nurse plans to take which best nursing action? 1. Call a family member to obtain telephone consent before the surgical procedure 2. Transport the client to the operating room immediately 3. Ask the hospital chaplain to sign the consent form 4. Call the nursing supervisor to initiate a court order for the surgical procedure Answer: 2 Rationale: Generally, there are only two instances in which the informed consent of an adult client is not needed. One instance is when an emergency is present and delaying treatment for the purpose of obtaining informed consent would result in injury or death to the client. The second instance is when the client waives the right to give informed consent. Test-Taking Strategy: Use the process of elimination. Note the key words surgery is required immediately. Option 3 can be easily eliminated first. Options 1 and 4 would delay treatment and should be eliminated. Review the issues surrounding informed consent if you had difficulty with this question. Level of Cognitive Ability: Application Client Needs: Safe, Effective Care Environment Integrated Process: Nursing Process/Implementation Content Area: Fundamental Skills Reference: Potter, P., & Perry, A. (2005). Fundamentals of nursing (6th ed.). St. Louis: Mosby, p. 417. 1633. A nurse is providing dietary instructions to a client with ulcerative colitis. Which of the following foods will the nurse instruct the client to avoid? 1. Whole grain cereals 2. Fresh corn on the cob 3. Bagels with cream cheese 4. Broiled chicken breast Answer: 2 Rationale: A low residue (low fiber) diet places less strain on the intestines because this type of diet is easier to digest. This diet is used for ulcerative colitis, diverticulitis, and irritable bowel syndrome. The item that contains the high residue and thus would place strain on the intestines is the fresh corn on the cob. Test-Taking Strategy: Note the key word avoid in the stem of the question. Note that the diagnosis in the question refers to an inflammation in the colon. With this in mind, you should easily be directed to option 2, the food item that would place the greatest strain on the intestinal tract. If you had difficulty with this question, review the diet prescribed for ulcerative colitis.

PN~CD~Questions~1601-1700 -

21 -

Level of Cognitive Ability: Application Client Needs: Physiological Integrity Integrated Process: Nursing Process/Implementation Content Area: Fundamental Skills Reference: Nix, S. (2005). Williams’ basic nutrition & diet therapy (11th ed.). St. Louis: Mosby, p. 334. 1634. A client has hyperphosphatemia. The nurse teaches the client to eliminate which of the following from the diet? 1. Coffee 2. Tea 3. Fish 4. Cocoa Answer: 3 Rationale: Foods that are naturally high in phosphates should be avoided by the client with hyperphosphatemia. These include fish, eggs, milk products, vegetables, whole grains, and carbonated beverages. Coffee, tea, and cocoa are not high in phosphates. Test-Taking Strategy: Use the process of elimination. Eliminate options 1, 2, and 4 because they are similar and contain caffeine. If this question was difficult, review the foods high in phosphate. Level of Cognitive Ability: Application Client Needs: Health Promotion and Maintenance Integrated Process: Teaching/Learning Content Area: Fundamental Skills References: Nix, S. (2005). Williams’ basic nutrition & diet therapy (11th ed.). St. Louis: Mosby, pp. 133-134. Peckenpaugh, N. (2003). Nutrition essentials and diet therapy (9th ed.). Philadelphia:W.B. Saunders, p. 98. Black, J., & Hawks, J. (2005). Medical-surgical nursing: Clinical management for positive outcomes (7th ed.). Philadelphia: W.B. Saunders, p. 242. 1635. A nurse manager asks a licensed practical nurse to work on her day off because of a short staffing problem. The licensed practical nurse has already made plans and does not want to work on the day scheduled to be off. The most assertive response by the licensed practical nurse to the nurse manager is which of the following? 1. “I can’t work that day.” 2. “You know how I hate to work extra shifts.” 3. “I will if you need me, but I might be a few minutes late.” 4. “I have planned to take the day off and will not be able to work on that day.” Answer: 4 Rationale: The most assertive response is the one that is direct and conveys a clear message in a positive manner. Option 1 is a passive response. Option 2 is an aggressive response. Option 3 is a passive-aggressive response. Test-Taking Strategy: Use the process of elimination focusing on the issue, the most assertive response. Note the relationship between the data in the question and option 4. Review assertive communication techniques if you had difficulty with this question.

PN~CD~Questions~1601-1700 -

22 -

Level of Cognitive Ability: Application Client Needs: Safe, Effective Care Environment Integrated Process: Communication and Documentation Content Area: Leadership/Management Reference: Potter, P., & Perry, A. (2005). Fundamentals of nursing (6th ed.). St. Louis: Mosby, pp. 418, 433. 1636. {PLACE FIGURE HERE (Fig. 19)} Ignatavicius, D., & Workman, M. (2006). Medical surgical nursing: Critical thinking for collaborative care (5th ed.). Philadelphia: W.B. Saunders, p. 939. Figure 44-13, A. 1636. A nurse is monitoring a client with a head injury and notes that the client is assuming this posture. The nurse notifies the registered nurse immediately to report that the client is exhibiting: 1. Decorticate posturing 2. Decerebrate posturing 3. Flaccid quadriplegia 4. Opisthotonos Answer: 1 Rationale: In decorticate posturing, the upper extremities (arms, wrists, and fingers) are flexed with adduction of the arms. The lower extremities are extended with internal rotation and plantar flexion. Decorticate posturing indicates a hemispheric lesion of the cerebral cortex. In decerebrate posturing, the upper extremities are stiffly extended and adducted with internal rotation and pronation of the palms. The lower extremities are stiffly extended with plantar flexion. The teeth are clenched, and the back is hyperextended. Decerebrate posturing indicates a lesion in the brainstem at the midbrain or upper pons. Flaccid quadriplegia is complete loss of muscle tone and paralysis of all four extremities, indicating a completely nonfunctional brainstem. Opisthotonos is prolonged arching of the back with the head and heels bent backward. Opisthotonos indicates meningeal irritation. Test-Taking Strategy: Note the position of the client’s arms and legs. Recalling that flexion of the upper extremities occurs in decorticate rigidity will direct you to option 1. Review abnormal postures if you had difficulty with this question. Level of Cognitive Ability: Analysis Client Needs: Physiological Integrity Integrated Process: Nursing Process/Data Collection Content Area: Adult Health/Neurological Reference: Ignatavicius, D., & Workman, M. (2006). Medical surgical nursing: Critical thinking for collaborative care (5th ed.). Philadelphia: W.B. Saunders, p. 939. {PLACE FIGURE HERE (Fig. 19). For Q#1636} Ignatavicius, D., & Workman, M. (2006). Medical surgical nursing: Critical thinking for collaborative care (5th ed.). Philadelphia: W.B. Saunders, p. 939. Figure 44-13, A. 1637. A nursing assistant who has been employed in a long-term care facility for 8 weeks is consistently 10 to 20 minutes late for work. The nurse’s lateness has caused unrest

PN~CD~Questions~1601-1700 -

23 -

with other staff members in the nursing unit. The nurse is due to receive a 3-month probation evaluation in 1 month. The nurse in charge of the nursing unit would most appropriately deal with this situation by: 1. Addressing the lateness with the nursing assistant at the 3-month probation evaluation 2. Telling the other staff members to cover for the nursing assistant until she arrives 3. Telling the nursing assistant that she will be fired if the behavior does not change 4. Confronting the nursing assistant to discuss the lateness and initiate problem-solving measures Answer: 4 Rationale: Arriving late to work is an unacceptable behavior. Although the nursing assistant’s behavior has caused unrest with other staff members, the primary concern is that this behavior affects client care. The nurse in charge needs to confront the nursing assistant and discuss the lateness and initiate problem-solving measures that ensure that the behavior does not continue. It is not appropriate to wait 1 month to address the behavior (option 1). It is also inappropriate to expect other staff members to cover for the nursing assistant until she arrives. Additionally, this action will increase the unrest with the staff members. Telling the nursing assistant that she will be fired if the behavior does not change does not provide confrontation or address problem-solving. However, firing may be an outcome if adequate warning has been issued and a change in behavior does not occur. Test-Taking Strategy: Use the process of elimination and the principles related to dealing with conflict and unacceptable behavior. Remember that it is most appropriate to confront and address issues if they occur. Review these principles if you had difficulty with this question. Level of Cognitive Ability: Application Client Needs: Safe, Effective Care Environment Integrated Process: Nursing Process/Implementation Content Area: Leadership/Management Reference: Zerwekh, J., & Claborn, J. (2003). Nursing today: Transitions and trends (4th ed.). Philadelphia: W.B. Saunders, pp. 168, 170. 1638. A nurse sees another nurse administer an incorrect medication to a client. The nurse that administered the incorrect medication does not report the error. The initial action by the nurse who observed the error is which of the following? 1. Ask the nurse if he or she intents to report the error 2. Contact the supervisor 3. Document the error in the client’s record 4. Complete an incident report Answer: 1 Rationale: The initial action by the nurse who observed the error would be to ask the nurse if he or she intends to report the error. To ensure client safety, all errors need to be reported. The client also needs to be assessed immediately. An incident report needs to be completed by the nurse who administered the incorrect medication. The appropriate documentation also needs to be made in the client’s record by the nurse who administered the incorrect medication. If the nurse who made the error indicates that the error will not be reported, then it may be necessary to contact the supervisor.

PN~CD~Questions~1601-1700 -

24 -

Test-Taking Strategy: Use the process of elimination noting the key words initial action. Eliminate options 3 and 4 first because they are similar and should be performed by the nurse who administered the incorrect medication. From the remaining options, focusing on the key words will direct you to option 1. Review the nursing responsibilities when a medication error occurs if you had difficulty with this question. Level of Cognitive Ability: Application Client Needs: Safe, Effective Care Environment Integrated Process: Nursing Process/Implementation Content Area: Leadership/Management Reference: Potter, P., & Perry, A. (2005). Fundamentals of nursing (6th ed.). St. Louis: Mosby, p. 841. 1639. A nurse is assisting in caring for a client with a head injury who is restless and is pulling at the intravenous (IV) line. The client’s physician does not want to sedate the client, and the family has requested that the client not be restrained. The nurse would most appropriately: 1. Ask a family member to sit with the client 2. Tell the family that the application of wrist restraints is critical to prevent injury to the client 3. Stay with the client and consult with the nurse manager about the situation 4. Ask a nursing assistant to monitor the client Answer: 3 Rationale: The nurse needs to stay with the client and consult with the nurse manager about the situation. It may be necessary for the nurse manager to contact the supervisor to obtain an additional staff member to care for the client. Since the client has a head injury, a major concern is the development of increased intracranial pressure (ICP). The application of restraints may agitate the client, causing further restlessness and thus increase ICP. A nursing assistant is not trained to monitor for increased ICP. It is inappropriate to ask a family member to sit with the client. Test-Taking Strategy: Use the process of elimination noting the key words most appropriately. Focusing on the data in the question will assist in eliminating option 2. Noting that the client has a head injury will assist in eliminating options 1 and 4. Review the nursing responsibilities when a client requires consistent monitoring if you had difficulty with this question. Level of Cognitive Ability: Application Client Needs: Safe, Effective Care Environment Integrated Process: Nursing Process/Implementation Content Area: Leadership/Management Reference: Potter, P., & Perry, A. (2005). Fundamentals of nursing (6th ed.). St. Louis: Mosby, p. 982. 1640. A vascular surgeon repeatedly asks a nurse to obtain signed consent forms on his surgical clients. The nurse is uncomfortable with obtaining the informed consents and explains these reasons to the surgeon, but the surgeon tells the nurse she will be reported if the consents are not obtained. The nurse would most appropriately manage this situation by:

PN~CD~Questions~1601-1700 -

25 -

1. Obtaining the informed consents 2. Contacting the national medical association and reporting the surgeon 3. Stating to the surgeon, “I don’t really care if you report me; I am not obtaining the consents.” 4. Discussing the situation with the nurse manager Answer: 4 Rationale: If a conflict arises, it is most appropriate to try to resolve the conflict directly. In this situation, the nurse has attempted to explain the reasons for being uncomfortable with the surgeon but was unable to resolve the conflict. The nurse would then most appropriately use the organizational channels of communication and discuss the issue with the nurse manager who would then proceed to resolve the conflict. The nurse manager may attempt to discuss the situation with the surgeon or seek assistance from the nursing supervisor. Options 1, 2, and 3 are inappropriate actions. Option 1 ignores the issue. Option 2 is inappropriate because the nurse needs to use the appropriate organizational channels of communication to resolve the conflict. Option 3 is an inappropriate statement and will initiate further conflict between the nurse and surgeon. Test-Taking Strategy: Use the process of elimination. Eliminate option 1 because it ignores the issue. Next, eliminate option 3 because this statement will initiate further conflict between the nurse and surgeon. From the remaining options, select option 4 because it identifies appropriate use of the organizational channels of communication. Review the principles related to managing conflicts if you had difficulty with this question. Level of Cognitive Ability: Application Client Needs: Safe, Effective Care Environment Integrated Process: Nursing Process/Implementation Content Area: Leadership/Management References: Potter, P., & Perry, A. (2005). Fundamentals of nursing (6th ed.). St. Louis: Mosby, p. 440. Zerwekh, J., & Claborn, J. (2003). Nursing today: Transitions and trends (4th ed.). Philadelphia: W.B. Saunders, pp. 166-168. 1641. A child with cerebral palsy (CP) is working to achieve maximal potential for locomotion, self-care, and socialization in school. The nurse would work with the child to meet these goals by: 1. Keeping the child in a special education classroom with other children with similar disabilities 2. Lying the child in the supine position with a 30-degree elevation of the head to facilitate feeding 3. Removing ankle-foot orthoses and braces once the child arrives at school 4. Placing the child on a wheeled scooter board Answer: 4 Rationale: Option 4 provides the child with maximal potential in locomotion, self-care, and socialization. The child can move around independently on his or her abdomen anywhere the child wants to go and can interact with others as desired. Orthoses must be used all the time to aid locomotion (option 3). Option 1 does not provide for maximal socialization and normalization; rather, children with CP need to be mainstreamed as

PN~CD~Questions~1601-1700 -

26 -

much as cognitively able. Not all children with CP are intellectually challenged. Option 2 does not provide for normalization in self-care. Just as children without CP sit up and use assistive devices when eating, so should children with CP. Test-Taking Strategy: Use the process of elimination. Noting the issue, achieve maximal potential for locomotion, self-care, and socialization, will direct you to option 4. Review care to the child with CP if you had difficulty with this question. Level of Cognitive Ability: Application Client Needs: Psychosocial Integrity Integrated Process: Nursing Process/Implementation Content Area: Child Health Reference: Wong, D., & Hockenberry, M. (2003). Wong’s nursing care of infants and children (7th ed.). St. Louis: Mosby, p. 1838. 1642. A nurse in charge of a nursing unit in a long-term care facility is concerned because staff members openly verbalize racial comments about clients on the unit. The nurse would most appropriately manage this concern by: 1. Ignoring the racial comments 2. Leaving articles about racial prejudice in the nurse’s lounge 3. Reporting the racial comments to the grievance committee 4. Discouraging the racial comments Answer: 4 Rationale: Prejudice reduction is a method of managing or discouraging racial comments made by others. When racial comments are discouraged, fewer comments will be made. Ignoring the racial comments is an inappropriate option because the concern will not addressed. Leaving articles about racial prejudice in the nurse’s lounge indirectly addresses the issue. Additionally the nurse can not ensure that the staff will read the articles. Likewise, reporting the racial comments to the grievance committee does not directly address the issue. The best approach that the nurse would take is to directly discuss the concern with the staff members. This action is not identified in the options. Therefore from the options presented, option 4 would most appropriately manage this concern. Test-Taking Strategy: Use the process of elimination. Eliminate options 1, 2, and 3 because they are similar and do not directly address the issue of concern. Review the methods of discouraging and preventing prejudices if you had difficulty with this question. Level of Cognitive Ability: Application Client Needs: Safe, Effective Care Environment Integrated Process: Nursing Process/Implementation Content Area: Leadership/Management Reference: Potter, P., & Perry, A. (2005). Fundamentals of nursing (6th ed.). St. Louis: Mosby, pp. 120-121. 1643. A client receives meperidine (Demerol) by the intramuscular route. Thirty minutes after receiving the medication, signs of an allergy to the medication develop in the client. The client’s temperature is 101° F, and the client’s skin is warm and flushed with a notable rash on the chest and back. The nurse further assesses the client and contacts the

PN~CD~Questions~1601-1700 -

27 -

registered nurse who then contacts the physician. The nurse documents which of the following on the incident report? 1. The client had an allergy to the meperidine 2. The physician was notified because a rash developed in the client after receiving meperidine 3. Thirty minutes after receiving meperidine, the temperature is 101° F, skin is warm and flushed, and a rash is noted on the chest and back; the physician was notified 4. The client apparently is allergic to meperidine as noted by a temperature of 101° F, warm and flushed skin, and a rash on the chest and back Answer: 3 Rationale: The nurse should document relevant information in an accurate, complete, and objective form. Option 1 does not identify objective data. Option 4 makes an interpretation about the occurrence. Although option 2 identifies accurate data, it is incomplete. Test-Taking Strategy: Use the process of elimination recalling that documentation needs to include relevant information in an accurate, complete, and objective form. This guideline will assist in eliminating options 1, 2, and 4. Also note the relationship of the data in the question and in the correct option. Review the principles related to documentation if you had difficulty with this question. Level of Cognitive Ability: Application Client Needs: Safe, Effective Care Environment Integrated Process: Communication and Documentation Content Area: Fundamental Skills Reference: Potter, P., & Perry, A. (2005). Fundamentals of nursing (6th ed.). St. Louis: Mosby, pp. 481-483. 1644. A client experiences subinvolution during the puerperium. The nurse recalls that which of the following are the most common causes for this occurrence? 1. Maternal hypertension and infection 2. Afterpains and increased estrogen levels 3. Increased estrogen and progesterone levels 4. Retained placental fragments and infections Answer: 4 Rationale: Retained placental fragments and infections are the primary causes of subinvolution. When either of these processes is present, the uterus has difficulty contracting. Test-Taking Strategy: The key words in the stem of the question are most common causes. Remember that retained placental fragments and infections are the primary causes of subinvolution. Review the causes of this occurrence if you had difficulty with this question. Level of Cognitive Ability: Comprehension Client Needs: Physiological Integrity Integrated Process: Nursing Process/Data Collection Content Area: Maternity/Postpartum Reference: Leifer, G. (2005). Maternity nursing (9th ed.). Philadelphia: W.B. Saunders, pp. 291-292.

PN~CD~Questions~1601-1700 -

28 -

1645. A nurse who works in a cardiac unit reports to work and is told that she needs to float to the neurological nursing unit because of a short-staffing problem on that unit. The nurse reports to the unit and receives a client assignment for the day from the nurse manager. The nurse is angry with the assignment because she believes that the assignment is more difficult than the assignment delegated to other nurses on the unit. The nurse would most appropriately: 1. Tell the nurse manager to call the nursing supervisor 2. Refuse to do the assignment 3. Ask the nurse manager of the neurological unit to discuss the assignment 4. Return to the cardiac unit and discuss the assignment with the nurse manager on that unit Answer: 3 Rationale: If a nurse believes that the assignment is more difficult than the assignment delegated to other nurses on the unit, the nurse would most appropriately discuss the assignment with the nurse manager of the neurological unit. The nurse may or may not have a more difficult assignment than the other nursing staff. However, this action will assist in either identifying the rationale for the assignment or determining if the assignment is actually more difficult. A nurse would not refuse an assignment. Specific situations may be present in which a nurse should not take care of a specific client. An example of this type of situation may be if a pregnant nurse is assigned to care for a client with rubella or a client with an internal radiation implant. In these situations, the nurse would also discuss the assignment with the nurse manager. The nurse would not return to the cardiac unit. This action indicates client abandonment. Additionally, this action does not address the conflict directly. Option 1 is an aggressive action and does not address the conflict directly. Test-Taking Strategy: Use the process of elimination noting the key words most appropriately. Eliminate option 2 first because a nurse would not refuse an assignment. Next, eliminate option 4 because this action indicates client abandonment. From the remaining options, select option 3 because it directly deals with the conflict. Review the appropriate methods of dealing with a conflict if you had difficulty with this question. Level of Cognitive Ability: Application Client Needs: Safe, Effective Care Environment Integrated Process: Nursing Process/Implementation Content Area: Leadership/Management Reference: Potter, P., & Perry, A. (2005). Fundamentals of nursing (6th ed.). St. Louis: Mosby, pp. 378-378, 418-419. 1646. A nurse is caring for a hospitalized child newly diagnosed with type 1 diabetes mellitus. At 11:00 AM, the child suddenly complains of weakness, headache, and blurred vision. The nurse would immediately: 1. Give the child ½ cup of orange juice to drink 2. Obtain a blood glucose reading 3. Call the dietary department and ask that the lunch tray be delivered early 4. Contact the physician Answer: 2

PN~CD~Questions~1601-1700 -

29 -

Rationale: The signs of hypoglycemia and hyperglycemia may be difficult to distinguish. Weakness, headache, and blurred vision can occur in either blood glucose alteration. A blood glucose reading will assist in confirming the diagnosis so that the appropriate action can be taken. Option 1 would be implemented if the child had hypoglycemia. Option 3 is inappropriate because the child should eat meals at basically the same time each day to achieve the best diabetic control. Contacting the physician would not be the immediate action; however, the nurse would inform the registered nurse of the situation. Test-Taking Strategy: Use the process of elimination noting the key word immediately. Note the signs identified in the question and that the child is hospitalized. Recalling that the signs of hypoglycemia and hyperglycemia may be difficult to distinguish will direct you to option 2. Review the immediate treatment measures for hypoglycemia and hyperglycemia if you had difficulty with this question. Level of Cognitive Ability: Application Client Needs: Safe, Effective Care Environment Integrated Process: Nursing Process/Implementation Content Area: Child Health References: Price, D., & Gwin, J. (2005). Thompson’s pediatric nursing (9th ed.). Philadelphia: W.B. Saunders, p. 285. Wong, D., & Hockenberry, M. (2003). Wong’s nursing care of infants and children (7th ed.). St. Louis: Mosby, p. 1740. 1647. A nurse determines that a child with type 1 diabetes mellitus is having a hypoglycemic reaction. The nurse should give the child which of the following to treat the reaction? 1. One sugar cube 2. A teaspoon of sugar 3. One-half cup of diet cola 4. One-half cup of fruit juice Answer: 4 Rationale: Hypoglycemia is immediately treated with 10 to 15 g of carbohydrate. Glucose tablets or glucose gel may be administered. Other items used to treat hypoglycemia include ½ cup of fruit juice, ½ cup of regular (nondiet) soft drink, 8 oz of skim milk, 6 to 10 hard candies, 4 cubes of sugar or 4 tsp of sugar, 6 saltines, 3 graham crackers, or 1 tbsp of honey or syrup. The items in options 1, 2, and 3 would not adequately treat hypoglycemia. Test-Taking Strategy: Use the process of elimination. Eliminate options 1 and 2 first because they are similar. From the remaining options, select option 4 because a diet cola does not contain the adequate amount of carbohydrate needed to treat hypoglycemia. Review the treatment for hypoglycemia if you had difficulty with this question. Level of Cognitive Ability: Application Client Needs: Safe, Effective Care Environment Integrated Process: Nursing Process/Implementation Content Area: Child Health References: Price, D., & Gwin, J. (2005). Thompson’s pediatric nursing (9th ed.). Philadelphia: W.B. Saunders, p. 285. Wong, D., & Hockenberry, M. (2003). Wong’s nursing care of infants and children (7th

PN~CD~Questions~1601-1700 -

30 -

ed.). St. Louis: Mosby, p. 1740. 1648. A mother of a 6-year-old-child calls a nurse who lives in the neighborhood and tells the nurse that her child accidentally rubbed waterproof sunscreen in his eyes. The nurse would tell the mother to immediately: 1. Flush the child’s eyes for 15 minutes with water 2. Have the child wipe the eyes with a wet towel 3. Tell the child to blink continuously to get the sunscreen out of the eye 4. Call the poison control center Answer: 4 Rationale: Waterproof sunscreen should never be placed near the eyes. Waterproof sunscreen causes severe pain and a chemical burn that can damage the child’s vision. Flushing the eyes with water does not stop the burning. The mother should be instructed to call the poison control center and to take the child to the emergency room. Special chemicals will be needed to flush the sunscreen out of the eyes and preserve vision. Wiping the eyes will increase the pain and burning. Blinking will not alleviate the pain or remove the sunscreen from the eyes. Test-Taking Strategy: Use the process of elimination and guidelines related to treating poisoning. Remember that the best immediate action is to call the poison control center. Review the principles related to poisoning if you had difficulty with this question. Level of Cognitive Ability: Application Client Needs: Safe, Effective Care Environment Integrated Process: Nursing Process/Implementation Content Area: Child Health Reference: Wong, D., & Hockenberry, M. (2003). Wong’s nursing care of infants and children (7th ed.). St. Louis: Mosby, p. 669. 1649. A mother of a 9-year-old child calls the emergency room and tells the nurse that her child received a minor burn on the hand after accidentally touching a grill during a family cookout. The mother asks the nurse for advice on how to treat the burn. The nurse tells the mother to immediately: 1. Apply a tepid compress to the child’s hand 2. Apply an ice pack to the child’s hand 3. Place the child’s hand under cool running water 4. Apply a sterile bandage tightly over the burn area to prevent swelling Answer: 3 Rationale: Most minor burns can be handled at home by the parents. For minor burns, exposure to cool running water is the best treatment. This stops the burning process and helps to alleviate pain. Ice is contraindicated because it may add more damage to already injured skin. Option 4 is an incorrect measure. Additionally the mother may not have a sterile bandage available. Test-Taking Strategy: Use the process of elimination. Eliminate option 4 first because this is an incorrect measure and because there is no data in the question that indicates that an open wound exists. Eliminate option 2 because the temperature of ice is extreme and could cause further damage. From the remaining options, use the principles related to the effects of tepid and cool temperatures. This will direct you to option 3. Review the

PN~CD~Questions~1601-1700 -

31 -

immediate treatment measures for minor burns if you had difficulty with this question. Level of Cognitive Ability: Application Client Needs: Safe, Effective Care Environment Integrated Process: Nursing Process/Implementation Content Area: Child Health Reference: Wong, D., & Hockenberry, M. (2003). Wong’s nursing care of infants and children (7th ed.). St. Louis: Mosby, p. 1235. 1650. A male adult calls the emergency room and tells the nurse that he received a bee sting to the arm while weeding a garden. The client states that he has received bee stings in the past and is not allergic to bees. The client states that the site is painful and asks the nurse for advice to alleviate the pain. The nurse tells the client to first: 1. Take two acetaminophen (Tylenol) 2. Place a heating pad to the site 3. Apply ice and elevate the site 4. Lie down and elevate the arm Answer: 3 Rationale: When a bee sting occurs and is painful, it is best to treat the site locally rather than systemically. Pain may be alleviated by the application of an ice pack and elevating the site. A heating pad will increase discomfort at the site. Acetaminophen may be taken by the client to assist in alleviating discomfort, but this would not treat the injury at a local level. Lying down and elevating the arm may have some effect on reducing edema at the site, but will not directly assist in alleviating the pain at the site of injury. Test-Taking Strategy: Use the process of elimination noting the key word first. Focus on the issue, that the site of the bee sting is painful. Eliminate option 1 because this measure produces a systemic not a local effect. Eliminate option 4 next because this measure will not directly assist in alleviating the pain at the site of injury. From the remaining options, recalling the effects of heat will assist in eliminating option 2 and direct you to option 3. Review the initial measures that will alleviate pain from a bee sting if you had difficulty with this question. Level of Cognitive Ability: Application Client Needs: Safe, Effective Care Environment Integrated Process: Nursing Process/Implementation Content Area: Adult Health/Immune References: Ignatavicius, D., & Workman, M. (2006). Medical-surgical nursing: Critical thinking for collaborative care (5th ed.). Philadelphia: W.B. Saunders, p. 183. Lewis, S., Heitkemper, M., & Dirksen, S. (2004). Medical-surgical nursing: Assessment and management of clinical problems (6th ed.). St. Louis: Mosby, p. 498. 1651. A nurse responds to an external disaster that occurred in a large city when a building collapsed. There are numerous victims who require treatment. Which victim will the nurse attend to first? 1. An alert victim who has numerous bruises on the arms and legs 2. A victim with a partial amputation of a leg who is bleeding profusely 3. A hysterical victim who received a head injury 4. A victim who sustained multiple serious injuries and is deceased

PN~CD~Questions~1601-1700 -

32 -

Answer: 2 Rationale: The nurse determines which victim will be attended to first based on the seriousness of the injury sustained by the victims involved in the disaster. The victim who must be treated immediately otherwise, life, limb, or vision will be threatened is categorized as emergent and is the priority (option 2). The victim who requires treatment, but life, limb, or vision is not threatened if care can be provided within 1 to 2 hours is considered urgent and is the second priority (option 3). If the victim requires evaluation and possible treatment, but time is not a critical factor, this is categorized as nonurgent and is the third priority (option 1). In such a disaster, the victim that sustained multiple serious injuries and is deceased will not be the priority. Test-Taking Strategy: Use the process of elimination and prioritization skills. Noting the key words bleeding profusely in option 2 will direct you to this option. Review the principles related to triage if you had difficulty with this question. Level of Cognitive Ability: Application Client Needs: Safe, Effective Care Environment Integrated Process: Nursing Process/Implementation Content Area: Delegating/Prioritizing Reference: Black, J., & Hawks, J. (2005). Medical-surgical nursing: Clinical management for positive outcomes (7th ed.). Philadelphia: W.B. Saunders, pp. 2484-2485. 1652. A nurse performs a vaginal assessment on a pregnant client in labor and notes the presence of the umbilical cord protruding from the vagina. The nurse immediately: 1. Places a gloved hand into the vagina and holds the presenting part off of the umbilical cord 2. Transports the client to the delivery room 3. Gently pushes the cord into the vagina 4. Summons for help from other staff members Answer: 1 Rationale: When cord prolapse occurs, prompt actions are taken to relieve cord compression and increase fetal oxygenation. The mother should be positioned with the hips higher than the head to shift the fetal presenting part toward the diaphragm. The nurse should also place a gloved hand into the vagina and hold the presenting part off of the umbilical cord. The nurse should summon for help, and other staff members should contact the physician and notify the delivery room. If the cord is protruding from the vagina, no attempt should be made to replace it because to do so could traumatize it and further reduce blood flow. Oxygen at 8 to 10 L/min by face mask is also administered to the mother to increase fetal oxygenation. Test-Taking Strategy: Use the process of elimination noting the key word immediately and focus on the issue that the umbilical cord is protruding from the vagina. Options 2 and 4 can be eliminated first because these actions delay necessary and immediate treatment. From the remaining options, recalling that the cord should not be pushed back into the vagina will direct you to option 1. Review priority nursing measures for prolapsed cord if you had difficulty with this question. Level of Cognitive Ability: Application Client Needs: Physiological Integrity

PN~CD~Questions~1601-1700 -

33 -

Integrated Process: Nursing Process/Implementation Content Area: Maternity/Intrapartum Reference: Leifer, G. (2005). Maternity nursing (9th ed.). Philadelphia: W.B. Saunders, p. 248. 1653. A postpartum nurse is collecting data from a client who delivered a viable newborn 2 hours ago. The nurse palpates the fundus and notes the character of the lochia. Which characteristic of the lochia would the nurse expect to note at this time? 1. White-colored lochia 2. Pink-colored lochia 3. Serosanguineous lochia 4. Dark red-colored lochia Answer: 4 Rationale: In assessment of the perineum, the lochia is checked for amount, color, and the presence of clots. The color of the lochia during the fourth stage of labor (the first 1 to 4 hours after birth) is a dark red color. Options 1, 2, and 3 are not the expected characteristics of lochia at this time period. Test-Taking Strategy: Use the process of elimination. Noting that the question refers to a client who delivered 2 hours ago will direct you to option 4. Review postpartum assessment findings if you had difficulty with this question. Level of Cognitive Ability: Analysis Client Needs: Health Promotion and Maintenance Integrated Process: Nursing Process/Data Collection Content Area: Maternity/Postpartum Reference: Lowdermilk, D., & Perry, S.E. (2004). Maternity & women’s health care (8th ed.). St. Louis: Mosby, p. 619. 1654. A postpartum nurse is caring for a mother following delivery of a newborn infant. The nurse performs a perineal assessment on the mother and notes a trickle of bright red blood coming from the perineum. The nurse checks the mother’s fundus and notes that it is firm. On review of the mother’s record, the nurse also notes that an episiotomy was performed. Based on this data, the nurse determines that: 1. This is a normal expectation following episiotomy 2. The perineal assessment should be performed more frequently 3. The bright red bleeding is abnormal and should be reported 4. The mother should be allowed bathroom privileges only

PN~CD~Questions~1601-1700 -

34 -

Answer: 3 Rationale: Lochial flow should be distinguished from bleeding originating from a laceration or episiotomy, which is usually brighter red than lochia, and appears as a continuous trickle of bleeding even though the fundus of the uterus is firm. This bright red bleeding is abnormal and needs to be reported. Options 1, 2, and 4 are incorrect interpretations of the assessment data. Test-Taking Strategy: Use the process of elimination. Regardless of the normal assessment data presented in the question, the key words in the question are bright red. This should be an indication that the vaginal flow is not normal. Review lochial flow and complications associated with episiotomy if you had difficulty with this question. Level of Cognitive Ability: Analysis Client Needs: Physiological Integrity Integrated Process: Nursing Process/Data Collection Content Area: Maternity/Postpartum Reference: Lowdermilk, D., & Perry, S.E. (2004). Maternity & women’s health care (8th ed.). St. Louis: Mosby, pp. 619, 629, 630. 1655. A nurse is caring for a client in labor. The nurse notes the presence of fetal bradycardia on the fetal monitor and suspects that the umbilical cord is compressed. The nurse immediately places the client in what position? 1. High-Fowler’s 2. Upright 3. With the hips elevated 4. Semi-Fowler Answer: 3 Rationale: When cord compression is suspected, the woman is immediately repositioned. The client may be turned from side to side or the hips are elevated to shift the fetal presenting part toward her diaphragm, thus relieving cord compression. A hands-andknees position may reduce compression on the cord that is entrapped behind the fetus. Several position changes may be required before the fetal pattern improves or resolves. Options 1, 2, and 4 are inappropriate positions and may cause further cord compression. Test-Taking Strategy: Focus on the issue, cord compression. Use the process of elimination. Eliminate options 1, 2, and 4 because these are similar positions. Also, visualizing the occurrence of cord compression will assist in eliminating these options. Review immediate nursing interventions if cord compression is suspected if you had difficulty with this question. Level of Cognitive Ability: Application Client Needs: Safe, Effective Care Environment Integrated Process: Nursing Process/Implementation Content Area: Maternity/Intrapartum Reference: McKinney, E., James, S., Murray, S., & Ashwill, J. (2005). Maternal-child nursing (2nd ed.). St. Louis: Elsevier, p. 696.

PN~CD~Questions~1601-1700 -

35 -

1656. A client who is breast-feeding her newborn infant is experiencing nipple soreness. To relieve the soreness, the nurse suggests that the client: 1. Avoid rotating breast-feeding positions 2. Stop nursing until the nipples heal 3. Substitute a bottle-feeding until the nipples heal 4. Position the infant with the ear, shoulder, and hip in straight alignment with the infant’s stomach against the mother Answer: 4 Rationale: The nurse would instruct the mother to position the infant with the ear, shoulder, and hip in straight alignment and with the infant’s stomach against the mother. Rotating breast-feeding positions, breaking suction with the little finger, nursing frequently, beginning feeding on the less sore nipple, not allowing the newborn to chew on the nipple or to sleep holding the nipple in the mouth, and applying tea bags soaked in warm water to the nipple are also measures to alleviate nipple soreness. Options 1, 2, and 3 are incorrect measures. Test-Taking Strategy: Use the process of elimination. Visualize each of the options in terms of how they may or may not lessen the nipple soreness. This will direct you to option 4. If you had difficulty answering the question, review these measures. Level of Cognitive Ability: Application Client Needs: Health Promotion and Maintenance Integrated Process: Nursing Process/Implementation Content Area: Maternity/Postpartum Reference: Lowdermilk, D., & Perry, S.E. (2004). Maternity & women’s health care (8th ed.). St. Louis: Mosby, p. 778. 1657. A nurse on the day shift receives a client assignment for the day. Which assigned client will the nurse assess first? 1. A client who had a pulmonary wedge resection 1 day ago and is attached to a closed chest tube drainage system 2. A client with ulcerative colitis who is scheduled to be discharged today 3. A client who was admitted during the night because of a severe exacerbation of asthma 4. A client scheduled for a kidney, ureter, and bladder (KUB) x-ray to determine the location of a kidney stone Answer: 3 Rationale: The nurse would first assess the client who was admitted during the night because of a severe exacerbation of asthma. This client’s problem directly relates to airway, and the nurse would need to determine that the interventions administered on admission and during the night were effective. The nurse would next assess the client who had a pulmonary wedge resection 1 day ago and is attached to a closed chest tube drainage system. This client’s problem also relates to airway; however, there is no indication that this client is experiencing any severe problems. The nurse would next assess the client scheduled for a KUB. The nurse would want to ensure that this client understands the reason for the x-ray. Additionally the nurse needs to determine if the client is experiencing any pain as a result of the kidney stone. The nurse would next assess the client preparing for discharge to determine the need for reinforcement of home

PN~CD~Questions~1601-1700 -

36 -

care instructions. Test-Taking Strategy: Use the process of elimination and the ABCs—airway, breathing, and circulation. This will assist in eliminating options 2 and 4. From the remaining options, noting the key words severe exacerbation in option 3 will direct you to this option. Review the guidelines related to assignment making if you had difficulty with this question. Level of Cognitive Ability: Application Client Needs: Safe, Effective Care Environment Integrated Process: Nursing Process/Implementation Content Area: Delegating/Prioritizing Reference: Potter, P., & Perry, A. (2005). Fundamentals of nursing (6th ed.). St. Louis: Mosby, pp. 318-320, 379. 1658. An emergency room nurse is caring for an older female client who may have been physically abused by her son. In planning care for the client, the priority nursing action is to: 1. Call the police 2. Tell the son that he cannot visit with his mother 3. Notify the social worker to investigate the situation 4. Obtain psychiatric help for the son Answer: 3 Rationale: If physical abuse or neglect is suspected, the priority nursing action is to assess the client, treat any physical injuries, and ensure that the client is safe. The nurse also notifies the physician and the social worker to investigate the situation. All states in the United States and other Western countries have laws requiring heath care professionals to report suspected elder abuse. Calling the police is a premature action. Telling the son that he cannot visit with his mother could initiate aggressive behavior in the son. Although the nurse may be involved in obtaining psychiatric assistance for the son, this is not the priority action. Test-Taking Strategy: Use the process of elimination noting the key word priority. Recalling the legal guidelines related to the issue of suspected abuse will direct you to option 3. Review care to the older client who is a victim of physical abuse if you had difficulty with this question. Level of Cognitive Ability: Application Client Needs: Safe, Effective Care Environment Integrated Process: Nursing Process/Planning Content Area: Delegating/Prioritizing References: Morrison-Valfre, M. (2005). Foundations of mental health care (3rd ed.). St. Louis: Mosby, p. 163. Potter, P., & Perry, A. (2005). Fundamentals of nursing (6th ed.). St. Louis: Mosby, p. 684. Stuart, G., & Laraia, M. (2005). Principles & practice of psychiatric nursing (8th ed.). St. Louis: Mosby, pp. 810-811. 1659. A nurse is assigned to care for a client being admitted to the nursing unit from the emergency room who attempted suicide by ingesting several sleeping pills. The nurse

PN~CD~Questions~1601-1700 -

37 -

implements which priority action when the client arrives to the unit? 1. Places the client on one-to-one suicide precautions 2. Places the client on suicide precautions with 15 minute checks 3. Asks the nursing assistant to check the client’s vital signs 4. Asks the client to report any suicidal thoughts immediately Answer: 1 Rationale: The nurse immediately implements one-to-one suicide precautions for the client who has attempted suicide. The nurse would next assess the client’s vital signs. The nurse may ask a nursing assistant to check the client’s vital signs; however, since the client ingested several sleeping pills, it is best for the nurse to perform the initial assessment. Option 4 may also be an appropriate nursing intervention, but the priority is identified in option 1. The priority intervention is constant supervision so that the nurse may intervene as needed if the client attempts to cause harm to self. Test-Taking Strategy: Use the process of elimination noting the key words attempted suicide. Option 1 is the only option that provides a safe environment. Review interventions for the suicidal client if you had difficulty with this question. Level of Cognitive Ability: Application Client Needs: Safe, Effective Care Environment Integrated Process: Nursing Process/Implementation Content Area: Delegating/Prioritizing Reference: Morrison-Valfre, M. (2005). Foundations of mental health care (3rd ed.). St. Louis: Mosby, p. 288. 1660. A woman is brought to the emergency room in a severe state of anxiety after witnessing a devastating car crash that killed two persons. A nurse assigned to care for the client would first: 1. Ask the client to talk to the police about what she witnessed 2. Take the client to a quiet room 3. Teach the client how to take deep breaths 4. Ask the client to describe the events of the car crash Answer: 2 Rationale: If a client with severe anxiety is left alone, he or she may feel abandoned and become overwhelmed. Placing the client in a quiet room is a priority intervention, but the nurse must stay with the client. It is not possible to teach the client deep breathing or relaxation exercises until the anxiety decreases. Asking the client to describe the events of the accident or to talk to the police should not be implemented until the anxiety has decreased. Test-Taking Strategy: Use the process of elimination noting the key word first. Eliminate options 1 and 4 first because they are similar and because these actions are not possible when the client is in a severe state of anxiety. From the remaining options, recall that it is not possible to teach the client deep breathing or relaxation exercises until the anxiety decreases. Review care to the client with severe anxiety if you had difficulty with this question. Level of Cognitive Ability: Application Client Needs: Psychosocial Integrity Integrated Process: Nursing Process/Implementation

PN~CD~Questions~1601-1700 -

38 -

Content Area: Mental Health Reference: Morrison-Valfre, M. (2005). Foundations of mental health care (3rd ed.). St. Louis: Mosby, p. 189. 1661. A female client in a manic state emerges from her room. The client is dressed in a low cut blouse and a miniskirt. She is not wearing underwear, and she proceeds to sit on a male client’s lap and begins to make sexual remarks and gestures to the male client. The nurse should take which action? 1. Tell the client that her privileges are suspended because of her behavior 2. Ask the client to go to her room and put some underwear on 3. Approach the client quietly, take her to her room, and assist her in getting dressed 4. Tell the client firmly to get off of the male client’s lap Answer: 3 Rationale: A person who is experiencing mania lacks insight and judgment, has poor impulse control, and is highly excitable. The nurse must take control without creating increased stress or anxiety to the client. A quiet, firm approach while distracting the client (walking her to her room and assisting her to get dressed) achieves the goal of having her dressed appropriately and preserving her psychosocial integrity. Options 1, 2, and 4 will increase the client’s anxiety. Test-Taking Strategy: Focus on the data provided in the question. Recalling that a primary nursing responsibility is to protect the client will direct you to option 3. Review care to the client with mania if you had difficulty with this question. Level of Cognitive Ability: Application Client Needs: Psychosocial Integrity Integrated Process: Nursing Process/Implementation Content Area: Mental Health References: Fortinash, K., & Holoday-Worret, P. (2005). Psychiatric mental health nursing (3rd ed.). St. Louis: Mosby, p. 221. Stuart, G., & Laraia, M. (2005). Principles & practice of psychiatric nursing (8th ed.). St. Louis: Mosby, p. 351. 1662. A nurse stops at the scene of an automobile crash to assist a victim. The victim complains of severe leg pain, is unable to get out of the automobile, and is frightened. The appropriate nursing action is to: 1. Stay with the victim 2. Assist the victim out of the automobile 3. Leave the victim to call an ambulance 4. Tell the victim to keep moving the leg to maintain circulation Answer: 1 Rationale: Since the victim complains of severe leg pain, a fracture should be suspected. With a suspected fracture, the victim is not moved unless it is dangerous to remain in that spot. The nurse should remain with the victim and have someone else call for emergency help. Before moving the client, the site of fracture is immobilized to prevent further injury. Moving the leg can cause further injury to the victim’s leg. Test-Taking Strategy: Use the process of elimination. Options 2 and 4 are eliminated first because either of these options could result in further injury to the victim. From the

PN~CD~Questions~1601-1700 -

39 -

remaining options, note that the victim is frightened. This will direct you to option 1. Review care to the victim with a fracture if you had difficulty with this question. Level of Cognitive Ability: Application Client Needs: Physiological Integrity Integrated Process: Nursing Process/Implementation Content Area: Fundamental Skills References: Black, J., & Hawks, J. (2005). Medical-surgical nursing: Clinical management for positive outcomes (7th ed.). Philadelphia: W.B. Saunders, p. 2501. Ignatavicius, D., & Workman, M. (2006). Medical-surgical nursing: Critical thinking for collaborative care (5th ed.). Philadelphia: W.B. Saunders, pp. 1192,1197. 1663. A client with skeletal traction applied to the right leg complains to the nurse of severe right leg pain in spite of being medicated with a prescribed analgesic. Which action should the nurse take? 1. Remove some of the traction weights 2. Provide pin care 3. Notify the registered nurse 4. Find out when the next dose of the prescribed analgesic can be given Answer: 3 Rationale: A client who complains of severe pain may need realignment or may have traction weights ordered that are too heavy. The nurse realigns the client, and if ineffective, then notifies the registered nurse who then calls the physician. The nurse never removes traction weights unless specifically prescribed by the physician. Severe leg pain, once traction has been established, indicates a problem. Medicating the client should be done after trying to determine and treat the cause. Providing pin care is unrelated to the problem as described. Test-Taking Strategy: Use the process of elimination. Note the key words severe pain and in spite of being medicated. Option 2 can be eliminated first because it is unrelated to the client’s problem. Next, eliminate option 1 because the nurse never removes the traction weights unless specifically prescribed by the physician. From the remaining options, noting that the client’s pain is severe will direct you to option 3. Review care to the client in traction if you had difficulty with this question. Level of Cognitive Ability: Application Client Needs: Physiological Integrity Integrated Process: Nursing Process/Implementation Content Area: Adult Health/Musculoskeletal Reference: Christensen, B., & Kockrow, E. (2003). Adult health nursing (4th ed.). St. Louis: Mosby, p. 153. 1664. A client with a left arm fracture complains of severe, diffuse pain that is unrelieved with pain medication. On further data collection, the nurse notes that the client experiences more pain during passive motion of the left arm as compared with active motion. Based on these findings, the nurse should take which action? 1. Check to see if it is time for more pain medication 2. Encourage the client to continue with active range-of-motion exercises to the left arm

PN~CD~Questions~1601-1700 -

40 -

3. Notify the registered nurse (RN) 4. Reassess the client in 30 minutes Answer: 3 Rationale: The client with early acute compartment syndrome typically complains of severe, diffuse pain that is unrelieved with pain medication. The client also complains that pain during passive motion is greater than during active motion. The nurse notifies the RN who then contacts the physician immediately. Options 1, 2, and 4 are inaccurate interventions. Test-Taking Strategy: Focus on the data presented in the question. Recall that these signs indicate early acute compartment syndrome. Remember that if this is suspected, the physician needs to be notified. Review the complications associated with a fracture of an extremity and the associated nursing interventions if you had difficulty with this question. Level of Cognitive Ability: Application Client Needs: Physiological Integrity Integrated Process: Nursing Process/Implementation Content Area: Adult Health/Musculoskeletal Reference: Christensen, B., & Kockrow, E. (2003). Adult health nursing (4th ed.). St. Louis: Mosby, pp. 143-144. 1665. A nurse who is assisting in caring for a client with a tracheostomy tube notes heavy bleeding from the stoma. The nurse also notes that the tracheostomy tube pulsates with the client’s heartbeat and suspects that the client has a trachea-innominate artery fistula. The nurse immediately prepares for which first action? 1. Transport the client to surgery 2. Remove the tracheostomy tube 3. Apply pressure to the innominate artery at the stoma site 4. Initiate an intravenous (IV) line Answer: 2 Rationale: With a trachea-innominate artery fistula, a malpositioned tube causes its distal tip to push against the lateral wall of the tracheostomy. Continued pressure from the tracheostomy tube causes necrosis and erosion of the innominate artery. This situation is a life-threatening complication. The tracheostomy tube is immediately removed. Direct pressure is then applied to the innominate artery at the stoma site. The client is then prepared for immediate surgical repair. An IV line will need to be initiated, but this is not the immediate action. Test-Taking Strategy: Use the process of elimination noting the key words immediately and first action. Focusing on the data in the question will assist in eliminating options 1 and 4. From the remaining options, recalling the cause of a trachea-innominate artery fistula will direct you to option 2 as the immediate action. Review the cause and management of a trachea-innominate artery fistula if you had difficulty with this question. Level of Cognitive Ability: Application Client Needs: Physiological Integrity Integrated Process: Nursing Process/Implementation Content Area: Delegating/Prioritizing

PN~CD~Questions~1601-1700 -

41 -

Reference: Black, J., & Hawks, J. (2005). Medical-surgical nursing: Clinical management for positive outcomes (7th ed.). Philadelphia: W.B. Saunders, p. 1779. 1666. A nurse is caring for a client who had a tracheostomy tube inserted 1 week ago. The client begins to cough vigorously, and accidental decannulation of the tracheostomy tube occurs. The nurse immediately plans to: 1. Replace the tracheostomy tube 2. Call an anesthesiologist 3. Call a respiratory therapist 4. Call a code Answer: 1 Rationale: If decannulation of a tracheostomy tube occurs 72 hours after surgical placement of the tracheostomy, the nurse extends the client’s neck and opens the tissue of the stoma to secure an airway. With the obturator inserted into the tracheostomy tube, the nurse quickly and gently replaces the tube and removes the obturator. The nurse checks for airflow through the tube and for bilateral breath sounds. If unable to secure the airway, the nurse notifies a more experienced nurse, respiratory therapist, or physician for assistance and attempts to ventilate the client with a bag-valve mask. If the client is in distress and further attempts to secure the airway fail, the nurse calls the resuscitation team, including an anesthesiologist, for assistance and calls a code if necessary. Test-Taking Strategy: Use the process of elimination noting the key word immediately. Focus on the issue, accidental decannulation of the tracheostomy tube, and visualize this occurrence. Remember that when decannulation occurs, the tracheostomy tube needs to be immediately replaced. Review care to the client when accidental decannulation occurs if you had difficulty with this question. Level of Cognitive Ability: Application Client Needs: Physiological Integrity Integrated Process: Nursing Process/Implementation Content Area: Delegating/Prioritizing Reference: Christensen, B., & Kockrow, E. (2003). Foundations of nursing (4th ed.). St. Louis: Mosby, p. 459. 1667. A nurse is suctioning an adult client through a tracheostomy tube. During the procedure, the nurse notes that the client’s oxygen saturation by pulse oximetry is 89%. The nurse would: 1. Continue suctioning 2. Terminate the suctioning procedure 3. Obtain a smaller sized suction catheter 4. Call respiratory therapy Answer: 2 Rationale: The nurse should monitor the client’s heart rate and pulse oximetry during suctioning to assess the client’s tolerance of the procedure. Oxygen desaturation below 90% indicates hypoxemia. If hypoxia occurs during suctioning, the nurse terminates the suctioning procedure. Using the 100% oxygen delivery system, the client is reoxygenated until baseline values are achieved. The size of the catheter should not

PN~CD~Questions~1601-1700 -

42 -

exceed half of the size of the tracheal lumen. In adults the standard catheter size is 12 to 14 French. Adequate catheter size facilitates efficient removal of secretions without causing hypoxemia. Test-Taking Strategy: Use the process of elimination. Noting that the client’s pulse oximetry is 89% and recalling the complications associated with suctioning will direct you to option 2. Review the complications associated with suctioning and the appropriate nursing interventions if you had difficulty with this question. Level of Cognitive Ability: Application Client Needs: Physiological Integrity Integrated Process: Nursing Process/Implementation Content Area: Adult Health/Respiratory Reference: Christensen, B., & Kockrow, E. (2003). Foundations of nursing (4th ed.). St. Louis: Mosby, p. 458. 1668. A nurse reinforces instructions to a breast-feeding mother who is experiencing breast engorgement about measures that will provide comfort. Which statement by the mother indicates an understanding of these measures? 1. “I won’t breast-feed during the nighttime hours.” 2. “I will take a cool shower just before I breast-feed.” 3. “I will massage the breasts before feeding to stimulate let-down.” 4. “I will avoid wearing a bra.” Answer: 3 Rationale: Comfort measures for breast engorgement include: massaging the breasts before feeding to stimulate let-down, wearing a supportive well-fitting bra at all times, taking a warm shower just before feeding or applying warm compresses, and alternating the breasts during feeding. Options 1, 2, and 4 are incorrect measures. Test-Taking Strategy: Focus on the issue, breast engorgement. Use the process of elimination and visualize each of the descriptions in the options to assist in identifying the correct measure. If you had difficulty answering the question, review these measures. Level of Cognitive Ability: Comprehension Client Needs: Health Promotion and Maintenance Integrated Process: Nursing Process/Evaluation Content Area: Maternity/Postpartum Reference: Lowdermilk, D., & Perry, S.E. (2004). Maternity & women’s health care (8th ed.). St. Louis: Mosby, p. 777. 1669. A nurse is caring for a client with a closed chest tube drainage system. When repositioning the client, the chest tube disconnects. The immediate nursing action is to: 1. Clamp the chest tube 2. Reattach the chest tube to the drainage system 3. Call the physician 4. Instruct the client to inhale Answer: 2 Rationale: In most situations, clamping of chest tubes is contraindicated, and agency policy and procedure must be followed with regard to clamping a chest tube. When the client has a residual air leak or pneumothorax, clamping the chest tube may precipitate a

PN~CD~Questions~1601-1700 -

43 -

tension pneumothorax because the air has no escape route. If the tube becomes disconnected, it is best to immediately reattach it to the drainage system or to submerge the end in a bottle of sterile water or saline to reestablish a water seal. If sterile water or saline is not readily available, it is preferable to leave the tube open because the risk of tension pneumothorax outweighs the consequences of an open tube. The nurse would also notify the registered nurse of the occurrence. The physician may need to be notified, but this is not the immediate action. The client would not be instructed to inhale. Test-Taking Strategy: Use the process of elimination noting the key word immediate. Focus on the issue that the chest tube disconnects. It would seem reasonable that if a chest tube disconnects, the immediate action would be to reattach the chest tube to the drainage system. Review the immediate nursing actions related to the complications associated with a closed chest tube drainage system if you had difficulty with this question. Level of Cognitive Ability: Application Client Needs: Physiological Integrity Integrated Process: Nursing Process/Implementation Content Area: Adult Health/Respiratory Reference: Christensen, B., & Kockrow, E. (2003). Adult health nursing (4th ed.). St. Louis: Mosby, pp. 384-385. 1670. A nurse is assisting a client with a closed chest tube drainage system to get out of bed to a chair. During the transfer, the chest tube gets caught in the leg of the chair and accidentally dislodges from the insertion site. The immediate nursing action is to: 1. Transfer the client back to bed 2. Cover the insertion site with sterile gauze 3. Contact the physician 4. Reinsert the chest tube Answer: 2 Rationale: If a chest tube dislodges from the insertion site, the nurse immediately covers the site with sterile gauze. The nurse would then notify the registered nurse (RN) and perform a respiratory assessment on the client and assist the client back to bed. The RN would then contact the physician. The nurse does not reinsert a chest tube. The physician will reinsert the chest tube if necessary. Test-Taking Strategy: Use the process of elimination noting the key word immediate. Eliminate option 4 first because a nurse is not trained to insert a chest tube. From the remaining options, focusing on the issue, that the tube dislodges from the insertion site, and recalling the complications associated with this occurrence will direct you to option 2. Review the immediate nursing actions related to the complications associated with a closed chest tube drainage system if you had difficulty with this question. Level of Cognitive Ability: Application Client Needs: Physiological Integrity Integrated Process: Nursing Process/Implementation Content Area: Adult Health/Respiratory Reference: Black, J., & Hawks, J. (2005). Medical-surgical nursing: Clinical management for positive outcomes (7th ed.). Philadelphia: W.B. Saunders, p. 1866.

PN~CD~Questions~1601-1700 -

44 -

1671. The low-exhaled volume (low-pressure) alarm sounds on a ventilator. A nurse rushes to the client’s room and checks the client to determine the cause of the alarm, but is unable to do so. The immediate nursing action is to: 1. Call a code 2. Call the physician 3. Ventilate the client manually with a resuscitation bag 4. Call the respiratory therapist Answer: 3 Rationale: Mechanical ventilators have alarm systems that warn the nurse of a problem with either the client or the ventilator. Alarm systems must be activated and functional at all times. The nurse must recognize an emergency and intervene promptly so that complications are prevented. If the cause of an alarm cannot be determined, the nurse ventilates the client manually with a resuscitation bag until the problem is corrected by a second nurse, the respiratory therapist, or physician. The nurse also notifies the registered nurse (RN) of the occurrence and obtains assistance from the RN. Test-Taking Strategy: Use the process of elimination and note the key word immediate. Thinking about the purpose of the ventilator will easily direct you to option 3. Remember a ventilator provides oxygen to the client. The nurse needs to manually ventilate a client if an emergency such as this occurred. Also note that options 1, 2, and 4 are similar. Review care to the client on a mechanical ventilator if you had difficulty with this question. Level of Cognitive Ability: Application Client Needs: Physiological Integrity Integrated Process: Nursing Process/Implementation Content Area: Adult Health/Respiratory Reference: Linton, A., & Maebius, N. (2003). Introduction to medical-surgical nursing (3rd ed.). Philadelphia: W.B. Saunders, p. 473. 1672. A nurse is performing nasopharyngeal suctioning on a client and suddenly notes the presence of bloody secretions. The nurse would first: 1. Continue suctioning to remove the blood 2. Check the amount of suction pressure being applied 3. Remove the suction catheter from the nose and begin vigorous suctioning through the mouth 4. Encourage the client to cough out the bloody secretions Answer: 2 Rationale: The return of bloody secretions is an unexpected outcome related to suctioning. If this occurs, the nurse should first assess the client and then determine the amount of suction pressure being applied. The amount of suction pressure may need to be decreased. The nurse also needs to be sure that intermittent suction and catheter rotation is being done during suctioning. Continuing with the suctioning or vigorous suctioning through the mouth will cause increased trauma and thus increased bleeding. Suctioning is normally performed on clients who are unable to expectorate secretions. Therefore it is unlikely that the client will be able to cough out the bloody secretions. Test-Taking Strategy: Use the process of elimination. Eliminate options 1 and 3 first

PN~CD~Questions~1601-1700 -

45 -

because they are similar. Next, eliminate option 4 because it is unlikely that the client will be able to cough out the bloody secretions. Review immediate nursing actions when a complication occurs during suctioning if you had difficulty with this question. Level of Cognitive Ability: Application Client Needs: Physiological Integrity Integrated Process: Nursing Process/Implementation Content Area: Adult Health/Respiratory Reference: Potter, P., & Perry, A. (2005). Fundamentals of nursing (6th ed.). St. Louis: Mosby, p. 1108. 1673. A nurse is suctioning a client through a tracheostomy tube. During the procedure, the client begins to cough and the nurse notes the presence of an audible wheeze. The nurse attempts to remove the suction catheter from the client’s trachea, but is unable to do so. The nurse would first: 1. Call a code 2. Contact the physician 3. Administer a bronchodilator 4. Disconnect the suction source from the catheter Answer: 4 Rationale: The inability to remove a suction catheter is a critical situation. This finding along with the client’s symptoms presented in the question indicates the presence of bronchospasm and bronchoconstriction. The nurse would immediately disconnect the suction source from the catheter, but leave the catheter in the trachea. The nurse would then connect the oxygen source to the catheter. The nurse also notifies the registered nurse who then notifies the physician. The physician will most likely prescribe an inhaled bronchodilator. The nurse also prepares for emergency resuscitation if this situation occurs during suctioning. Test-Taking Strategy: Use the process of elimination noting the key word first. Eliminate option 3 first because this action requires a physician’s order. From the remaining options, visualize the situation presented in the question. Noting that the nurse is unable to remove the suction catheter from the client’s trachea will direct you to option 4. Review immediate nursing actions when a complication occurs during suctioning if you had difficulty with this question. Level of Cognitive Ability: Application Client Needs: Physiological Integrity Integrated Process: Nursing Process/Implementation Content Area: Adult Health/Respiratory Reference: Potter, P., & Perry, A. (2005). Fundamentals of nursing (6th ed.). St. Louis: Mosby, p. 1108. 1674. A client attached to mechanical ventilation suddenly becomes restless and pulls out the endotracheal tube. The nurse would first: 1. Check the client for spontaneous breathing 2. Prepare for reintubation 3. Call the physician 4. Call a code

PN~CD~Questions~1601-1700 -

46 -

Answer: 1 Rationale: If unexpected intubation occurs, the nurse would first check the client for airway patency, spontaneous breathing, and vital signs. The nurse would remain with the client, call for assistance from the registered nurse, and prepare for reintubation. There is no data in the question to indicate that a code needs to be called. Test-Taking Strategy: Note the key word first. Use the ABCs—airway, breathing, and circulation—to answer the question. This will direct you to option 1. Review the complications associated with an endotracheal tube and the associated nursing interventions if you had difficulty with this question. Level of Cognitive Ability: Application Client Needs: Physiological Integrity Integrated Process: Nursing Process/Implementation Content Area: Adult Health/Respiratory Reference: Potter, P., & Perry, A. (2005). Fundamentals of nursing (6th ed.). St. Louis: Mosby, p. 1116. 1675. A nurse checks a closed chest tube drainage system on a client who had a lobectomy 24 hours ago. The nurse notes that there has been no chest tube drainage for the past hour. The nurse would first: 1. Contact the registered nurse (RN) 2. Check the client’s blood pressure and heart rate 3. Check for kinks in the chest drainage system 4. Connect a new drainage system to the client’s chest tube Answer: 3 Rationale: If the nurse notes that a chest tube is not draining, the nurse would first check for a kink or possible clot in the chest drainage system. The nurse then notifies the RN and observes the client for respiratory distress or mediastinal shift (if this occurs, the physician is notified). Checking the heart rate and blood pressure is not directly related to the lack of chest tube drainage. Connecting a new drainage system to the client’s chest tube is done when the fluid drainage chamber is full. There is a specific procedure to follow when a new drainage system is connected to a client’s chest tube. Teat-Taking Strategy: Use the process of elimination noting the key word first. Focusing on the issue, that there has been no chest tube drainage, will direct you to option 3. Review the unexpected outcomes and related interventions in the care of a chest tube drainage system if you had difficulty with this question. Level of Cognitive Ability: Application Client Needs: Physiological Integrity Integrated Process: Nursing Process/Implementation Content Area: Adult Health/Respiratory Reference: Potter, P., & Perry, A. (2005). Fundamentals of nursing (6th ed.). St. Louis: Mosby, p. 1119. 1676. A cardiac monitor alarm sounds, and a nurse notes a straight line on the monitor screen. The immediate nursing action is to: 1. Call a code 2. Check the cardiac leads and wires

PN~CD~Questions~1601-1700 -

47 -

3. Confirm the rhythm 4. Assess the client Answer: 4 Rationale: If a monitor alarms sounds, the nurse should first assess the clinical status of the client to see if the problem is an actual dysrhythmia or a malfunction of the monitoring system. Asystole should not be mistaken for an unattached electrocardiogram wire. Options 1, 2, and 3 are not the first action. Test-Taking Strategy: Note the key word immediate. Use the steps of the nursing process, remembering that data collection is the first step. This will assist in eliminating options 1 and 2. From the remaining options, select option 4 because it is client focused. Remember to assess the client first. Review care to the client on a cardiac monitor if you had difficulty with this question. Level of Cognitive Ability: Application Client Needs: Physiological Integrity Integrated Process: Nursing Process/Implementation Content Area: Adult Health/Respiratory Reference: Black, J., & Hawks, J. (2005). Medical-surgical nursing: Clinical management for positive outcomes (7th ed.). Philadelphia: W.B. Saunders, p. 2469. 1677. A nurse notes that a client with acquired immunodeficiency syndrome (AIDS) appears anxious and is reluctant to ask questions. Which action would the nurse take first to best deal with these observations? 1. Minimize the time spent talking to the client 2. Ask a family member to be present when caring for the client 3. Discuss common fears and questions expressed by other clients with the same diagnosis 4. Ask the client why he or she is reluctant to ask questions Answer: 3 Rationale: Imparting the common fears and questions expressed by other clients often encourages the client to ask questions that were thought of but not spoken. The nurse should plan to spend additional time with the client. Requesting a family member to be present may reduce the client’s anxiety and may be tried after the nurse has talked to the client. However, communication with the client is needed to determine the source of the anxiety. Requesting an explanation is a nontherapeutic technique of communication. The client may not know the reason why he or she is reluctant to talk. Test-Taking Strategy: Use therapeutic communication techniques noting the key word first in the stem of the question. Remember to always address the client’s concerns and feelings. This will direct you to option 3. Review therapeutic communication techniques if you had difficulty with this question. Level of Cognitive Ability: Application Client Needs: Psychosocial Integrity Integrated Process: Nursing Process/Implementation Content Area: Fundamental Skills References: Christensen, B., & Kockrow, E. (2003). Adult health nursing (4th ed.). St. Louis: Mosby, p. 694.

48

PN~CD~Questions~1601-1700 -

-

Potter, P., & Perry, A. (2005), Fundamentals of nursing (6th ed.). St. Louis: Mosby, p. 437. 1678. A delivery room nurse performs an assessment on a mother who just delivered a healthy newborn infant. The nurse checks the uterine fundus expecting to note that it is positioned: 1. At the level of the umbilicus 2. To the left of the abdomen 3. Two fingerbreadths above the symphysis pubis 4. To the right of the abdomen Answer: 1 Rationale: Immediately after delivery, the uterine fundus should be at the level of the umbilicus or one to three fingerbreadths below it and in the midline of the abdomen. If the fundus is above the umbilicus, this may indicate that there are blood clots in the uterus that need to be expelled by fundal massage. A fundus that is not located in the midline may indicate a full bladder. Test-Taking Strategy: Focus on the issue, normal postdelivery assessment findings in the mother. Recalling the normal anatomy of the reproductive system will assist in directing you to option 1. If you had difficulty with this question, review postdelivery assessment findings. Level of Cognitive Ability: Comprehension Client Needs: Health Promotion and Maintenance Integrated Process: Nursing Process/Data Collection Content Area: Maternity/Intrapartum References: Lowdermilk, D., & Perry, S.E. (2004). Maternity & women’s health care (8th ed.). St. Louis: Mosby, p. 619. Murray, S., McKinney, E., & Gorrie, T. (2002). Foundations of maternal-newborn nursing (3rd ed.). Philadelphia: W.B. Saunders, p. 325. 1679. A physician places a Miller-Abbott tube in a client. Six hours later the nurse measures the length of the tube outside the nares and notes that the tube has advanced 6 cm since it was first placed. Based on this finding, which action would the nurse take next? 1. Notify the physician. 2. Initiate a tube feeding 3. Pull the tube out 6 cm and secure the tube to the nose with tape 4. Document the finding in the client’s record Answer: 4 Rationale: The Miller-Abbott tube is a nasoenteric tube, which is used to decompress the intestine and correct a bowel obstruction. Initial insertion of the tube is a physician’s responsibility. The tube is mercury weighted and either advances by gravity or may be advanced manually. Advancement of the tube can be monitored by measuring the tube and by taking serial x-rays. Options 1, 2, and 3 are incorrect nursing actions. The nurse would, however, keep the registered nurse informed about the progress with tube advancement. Test-Taking Strategy: Use the process of elimination noting the key word next.

PN~CD~Questions~1601-1700 -

49 -

Remembering that tube advancement by gravity is expected to occur with this type of tube will assist in directing you to the correct option. If you had difficulty with this question, review the function of this tube and the nursing care involved. Level of Cognitive Ability: Application Client Needs: Physiological Integrity Integrated Process: Communication and Documentation Content Area: Adult Health/Gastrointestinal Reference: Christensen, B., & Kockrow, E. (2003). Foundations of nursing (4th ed.). St. Louis: Mosby, p. 532. 1680. A postpartum nurse obtains the vital signs on a mother who delivered a healthy newborn infant 2 hours ago. The mother’s temperature is 100° F (38° C). The initial nursing action would be to: 1. Document the finding 2. Notify the physician 3. Administer acetaminophen (Tylenol) 4. Encourage oral fluid intake Answer: 4 Rationale: During the first 24 hours following delivery, the mother’s temperature may rise to 100° F (38° C) as a result of the dehydrating effects of labor. Therefore the initial nursing action is to encourage fluid intake. The nurse would document the temperature, but this is not the initial action. Options 2 and 3 are not necessary at this time. Test-Taking Strategy: Use the process of elimination. Note the key words 2 hours ago and initial nursing action. Recalling the effects of labor will direct you to option 4. Review normal vital signs following delivery and the appropriate nursing interventions if you had difficulty with this question. Level of Cognitive Ability: Application Client Needs: Health Promotion and Maintenance Integrated Process: Nursing Process/Implementation Content Area: Maternity/Postpartum References: Lowdermilk, D., & Perry, S.E. (2004). Maternity & women’s health care (8th ed.). St. Louis: Mosby, p. 617. Murray, S., McKinney, E., & Gorrie, T. (2002). Foundations of maternal-newborn nursing (3rd ed.). Philadelphia: W.B. Saunders, p. 440. 1681. A nurse is monitoring a client who is in the active stage of labor. The nurse notes a late deceleration on the fetal monitor. Based on this observation, the nurse immediately: 1. Administers oxygen via face mask to the mother 2. Documents the findings 3. Transports the client to the delivery room 4. Increases the rate of an oxytocin (Pitocin) infusion Answer: 1 Rationale: Late decelerations are due to uteroplacental insufficiency that occurs as a result of decreased blood flow and oxygen transfer to the fetus through the intervillous space during the uterine contractions. This causes hypoxemia; therefore oxygen is necessary. Late decelerations are considered an ominous sign, but do not necessarily

PN~CD~Questions~1601-1700 -

50 -

require immediate birth of the baby. The oxytocin infusion should be discontinued when a late deceleration is noted. The oxytocin would cause further hypoxemia because the medication stimulates contractions and leads to increased uteroplacental insufficiency. Although the finding needs to be documented, documentation is not the immediate action in this situation. Test-Taking Strategy: Note the key word immediately in the stem of the question. Use the ABCs—airway, breathing, and circulation—to direct you to option 1. Review the immediate nursing interventions when late decelerations occur if you had difficulty with this question. Level of Cognitive Ability: Application Client Needs: Physiological Integrity Integrated Process: Nursing Process/Implementation Content Area: Maternity/Intrapartum Reference: Leifer, G. (2005). Maternity nursing (9th ed.). Philadelphia: W.B. Saunders, p. 81. 1682. A client is brought to the labor unit and as the nurse is attaching the fetal heart monitor, the client’s membranes rupture spontaneously. The nurse immediately: 1. Monitors the contraction pattern 2. Checks the fetal heart rate (FHR) 3. Notes the amount, color, and odor of the amniotic fluid 4. Prepares the client for immediate delivery Answer: 2 Rationale: When the membranes rupture in the birth setting, the nurse immediately checks the FHR to detect changes associated with prolapse or compression of the umbilical cord. Options 1 and 3 may also be a component of care, but are not the immediate actions. There is no data to indicate that option 4 is necessary at this time. Test-Taking Strategy: Note the key word immediately in the stem of the question. Use the ABCs—airway, breathing, and circulation—to assist in answering the question. FHR is associated with fetal breathing and circulation. Review care to the client following a rupture of the membranes if you had difficulty with this question. Level of Cognitive Ability: Application Client Needs: Physiological Integrity Integrated Process: Nursing Process/Implementation Content Area: Maternity/Intrapartum Reference: Leifer, G. (2005). Maternity nursing (9th ed.). Philadelphia: W.B. Saunders, p. 248. 1683. A nurse is preparing to administer an injection of vitamin K to a newborn. When administering the injection, the nurse would select which injection site? 1. The lateral aspect of the middle third of the vastus lateralis muscle 2. The medial aspect of the upper third of the vastus lateralis muscle 3. The lower aspect of the rectus femoris muscle 4. The gluteal muscle Answer: 1 Rationale: The preferred injection site for vitamin K in the newborn is the lateral aspect

PN~CD~Questions~1601-1700 -

51 -

of the middle third of the vastus lateralis muscle in the newborn’s thigh. This muscle is the preferred injection site because it is free of major blood vessels and nerves and is large enough to absorb the medication. Options 2, 3, and 4 are incorrect injection sites. Test-Taking Strategy: Knowledge regarding the preferred injection site for a newborn is required to answer this question. Visualize the anatomical location of each of the areas in the options to direct you to option 1. If you had difficulty with this question, review the procedure for administering vitamin K in the newborn. Level of Cognitive Ability: Application Client Needs: Physiological Integrity Integrated Process: Nursing Process/Implementation Content Area: Maternity/Postpartum References: Lowdermilk, D., & Perry, S.E. (2004). Maternity & women’s health care (8th ed.). St. Louis: Mosby, p. 745. Murray, S., McKinney, E., & Gorrie, T. (2002). Foundations of maternal-newborn nursing (3rd ed.). Philadelphia: W.B. Saunders, p. 547. 1684. A nurse is giving a client a bed bath and drops the towel on the floor. The nurse should take which action? 1. Wash the hands, pick up the towel, and shake it off 2. Wash the hands and go to the linen room to obtain another towel 3. Borrow a towel from the client’s roommate 4. Use a bath blanket as a towel Answer: 2 Rationale: To prevent spreading the client’s germs, the nurse’s hands must be washed before leaving the client’s room. It is never appropriate to borrow other clients’ supplies because it is not consistent with general principles of infection control. It is not appropriate to use a bath blanket as a towel. The nurse would never use a supply that was dropped on the floor. Additionally, shaking linen spreads germs. Test-Taking Strategy: Use knowledge regarding infection control and the basic principles related to bathing a client to direct you to option 2. Review these basic principles if you had difficulty with this question. Level of Cognitive Ability: Application Client Needs: Safe, Effective Care Environment Integrated Process: Nursing Process/Implementation Content Area: Fundamental Skills Reference: Potter, P., & Perry, A. (2005). Fundamentals of nursing (6th ed.). St. Louis: Mosby, p. 1029. 1685. A child with pertussis (whooping cough) is being admitted to the pediatric unit. As soon as the child arrives to the unit, the nurse would first: 1. Place the child on a pulse oximeter 2. Weigh the child 3. Take the child’s temperature 4. Ask the parents about the child Answer: 1 Rationale: To adequately determine if the child is getting enough oxygen, the child is

PN~CD~Questions~1601-1700 -

52 -

placed on a pulse oximeter. The pulse oximeter will then provide ongoing information on the child’s oxygen level. The child is also immediately placed on a cardiorespiratory monitor to provide early identification of periods of apnea and bradycardia. The nurse would then perform an assessment, including taking the child’s temperature, weight, and asking the parents about the child. Test-Taking Strategy: Note the key word first. Focus on the child’s diagnosis and use the ABCs—airway, breathing, and circulation. This will direct you to option 1. Review the priority interventions in the care of a child with pertussis if you had difficulty with this question. Level of Cognitive Ability: Application Client Needs: Physiological Integrity Integrated Process: Nursing Process/Implementation Content Area: Delegating/Prioritizing Reference: Price, D., & Gwin, J. (2005). Thompson’s pediatric nursing (9th ed.). Philadelphia: W.B. Saunders, p. 255. 1686. A postpartum client asks a nurse when she can resume sexual activity. The appropriate nursing response is which of the following? 1. “Sexual activity can be resumed at any time.” 2. “Sexual activity can not be resumed until your 8-week physician checkup.” 3. “Sexual activity can be resumed in about 3 weeks when the episiotomy has healed, and the lochia has stopped.” 4. “Sexual activity can be resumed once you resume a normal menstrual period.” Answer: 3 Rationale: It is recommended that the woman refrain from sexual intercourse until the episiotomy has healed, and the lochia has stopped. This process usually takes about 3 weeks. Options 1, 2, and 4 are inaccurate. Test-Taking Strategy: Knowledge regarding instructions to the mother regarding sexual activity following delivery is required to answer this question. Noting the words “when the episiotomy has healed” in option 3 will direct you to this option. If you are unfamiliar with this information, review this content. Level of Cognitive Ability: Application Client Needs: Health Promotion and Maintenance Integrated Process: Nursing Process/Implementation Content Area: Maternity/Postpartum References: Lowdermilk, D., & Perry, S.E. (2004). Maternity & women’s health care (8th ed.). St. Louis: Mosby, p. 641. Murray, S., McKinney, E., & Gorrie, T. (2002). Foundations of maternal-newborn nursing (3rd ed.). Philadelphia: W.B. Saunders, p. 454. 1687. A licensed practical nurse (LPN) is monitoring a postoperative client on an hourly basis. The LPN notes that the client’s hourly urine output is 25 mL. Based on this finding, the first action of the LPN would be to: 1. Check the client’s overall intake and output record 2. Increase the rate of the IV fluid 3. Administer a 250-mL bolus of normal saline (0.9%)

PN~CD~Questions~1601-1700 -

53 -

4. Notify the registered nurse Answer: 1 Rationale: Clients are at risk for becoming hypovolemic after surgery, and often the first sign of hypovolemia is a decreasing urine output. However, the nurse needs additional data to make an accurate interpretation. Options 2 and 3 are not implemented without an order from the physician. The physician is called by the registered nurse (RN) once the LPN has gathered all necessary assessment data and has reported the information to the RN, including the overall fluid status and vital signs. Test-Taking Strategy: Note the key word first. Attempt to visualize the situation and use the steps of the nursing process to answer the question. Option 1 addresses the process of data collection. Additionally, eliminate options 2 and 3 because they require a physician’s order. Eliminate option 4 because in this situation, the LPN needs to gather additional information and report the information to the RN. Review the unexpected outcomes following surgery and the priority nursing interventions if you had difficulty with this question. Level of Cognitive Ability: Application Client Needs: Physiological Integrity Integrated Process: Nursing Process/Implementation Content Area: Fundamental Skills Reference: deWit, S. (2005). Fundamental concepts and skills for nursing. Philadelphia: W.B. Saunders, pp. 746, 748. 1688. A nurse is getting a postoperative client out of bed for the first time since surgery. The nurse raises the head of the bed, and the client complains of dizziness. Which of the following actions should the nurse take first? 1. Check the oxygen saturation level 2. Check the blood pressure 3. Lower the head of bed slowly until the dizziness is relieved 4. Have the client take some deep breaths Answer: 3 Rationale: Dizziness or feeling faint is not uncommon when a postoperative client is positioned upright for the first time after surgery. If this occurs, the nurse relieves the feeling by lowering the head of bed slowly until the dizziness is relieved. The nurse would then check the pulse and blood pressure. Since the problem is circulatory, not respiratory, options 1 and 4 are not the first actions to take. Test-Taking Strategy: Use the process of elimination noting the key word first. Note the relation between the issue in the question and option 3. Review the unexpected outcomes following surgery and the priority nursing interventions if you had difficulty with this question. Level of Cognitive Ability: Application Client Needs: Physiological Integrity Integrated Process: Nursing Process/Implementation Content Area: Delegating/Prioritizing Reference: deWit, S. (2005). Fundamental concepts and skills for nursing. Philadelphia: W.B. Saunders, p. 749.

PN~CD~Questions~1601-1700 -

54 -

1689. A hospitalized client with a history of angina pectoris is ambulating in the corridor. The client suddenly complains of severe substernal chest pain. The nurse should take which action first? 1. Administer sublingual nitroglycerin 2. Assist the client to sit or lie down 3. Apply nasal oxygen at a rate of 2 L/min 4. Check the client’s vital signs Answer: 2 Rationale: Chest pain is due to an imbalance between myocardial oxygen supply and demand. During episodes of pain, the nurse first limits the client’s activity and assists the client to a position of comfort, checks the vital signs, administers oxygen and medication according to protocol, and obtains a 12-lead electrocardiogram. Test-Taking Strategy: Use the process of elimination noting the key word first. Focus on the data in the question noting that the client is ambulating in the corridor. This will direct you to option 2. The client needs to stop activity and assume a position of comfort to reduce myocardial oxygen demand. Review the priority nursing actions when a client with angina experiences chest pain if you had difficulty with this question. Level of Cognitive Ability: Application Client Needs: Physiological Integrity Integrated Process: Nursing Process/Implementation Content Area: Delegating/Prioritizing References: Christensen, B., & Kockrow, E. (2003). Adult health nursing (4th ed.). St. Louis: Mosby, p. 313. Linton, A., & Maebius, N. (2003). Introduction to medical-surgical nursing (3rd ed.). Philadelphia: W.B. Saunders, p. 583. 1690. A client arrives to the emergency room with an episode of status asthmaticus. The nurse first: 1. Places the client in high Fowler’s position 2. Obtains a set of vital signs 3. Obtains equipment for starting an intravenous line 4. Administers oxygen at 21% Answer: 1 Rationale: The first nursing action is to place the client in a position that aids in respiration, which would be sitting bolt upright or in high Fowler’s. Other nursing actions follow in rapid sequence and include monitoring vital signs and administering bronchodilators and oxygen (but at levels of 2 to 5 L/min or 24% to 28% by ventimask). Insertion of an IV line and ongoing monitoring of respiratory status are also indicated. Test-Taking Strategy: Note the key word first. Eliminate option 4 first since oxygen at 21% is ambient air, not supplemental oxygen. Next, eliminate option 2 because it is an assessment and is not the best first action when a client is in severe respiratory distress. The correct option protects the client’s airway, which guides you to choose option 1 over option 3. Review the initial nursing actions in the care of a client with status asthmaticus if you had difficulty with this question. Level of Cognitive Ability: Application Client Needs: Physiological Integrity

PN~CD~Questions~1601-1700 -

55 -

Integrated Process: Nursing Process/Implementation Content Area: Adult Health/Respiratory Reference: Black, J., & Hawks, J. (2005). Medical-surgical nursing: Clinical management for positive outcomes (7th ed.). Philadelphia: W.B. Saunders, p. 1813. 1691. A client hospitalized with urolithiasis has a sudden notable decrease in urine output. The nurse would immediately: 1. Notify the registered nurse (RN) 2. Replace the Foley catheter with a new one 3. Tell the client to drink increased fluids 4. Obtain a urine specific gravity Answer: 1 Rationale: A sudden significant decrease in urine output, to either oliguria or anuria, represents obstruction of the urinary tract, usually at the bladder neck or urethra. This represents a medical emergency, requiring prompt treatment to preserve kidney function. In this instance, the nurse would notify the RN who would call the physician to report the findings immediately. There is no data in the question to indicate that a Foley catheter is present. Obtaining a urine specific gravity will not relieve the obstruction. Telling the client to increase fluid intake is incorrect. Additionally, if an obstruction is present, increasing fluids can cause hydronephrosis. Test-Taking Strategy: Use the process of elimination noting the key word immediately. Focus on the client’s diagnosis. Recalling that obstruction of the urinary tract can occur and that a sudden decrease in urine output signals the presence of obstruction will direct you to option 1. Review the complications of urolithiasis and the associated immediate nursing interventions if you had difficulty with this question. Level of Cognitive Ability: Application Client Needs: Physiological Integrity Integrated Process: Nursing Process/Implementation Content Area: Adult Health/Renal Reference: Linton, A., & Maebius, N. (2003). Introduction to medical-surgical nursing (3rd ed.). Philadelphia: W.B. Saunders, p. 772. 1692. A nurse enters the room of a client with type 1 diabetes mellitus and finds the client difficult to arouse. The client’s skin is warm and flushed, and the pulse and respiratory rate are elevated from the client’s baseline. The nurse would immediately: 1. Prepare for the administration of an insulin drip 2. Give the client a glass of orange juice 3. Prepare for the administration of a bolus dose of 50% dextrose 4. Check the client’s capillary blood glucose Answer: 4 Rationale: The client’s signs and symptoms are consistent with hyperglycemia. The nurse must first obtain a blood glucose reading to verify this interpretation, which would then be reported to the physician for subsequent orders. Options 2 and 3 would be implemented as needed in the treatment of hypoglycemia. Insulin therapy is guided by blood glucose measurement.

PN~CD~Questions~1601-1700 -

56 -

Test-Taking Strategy: Use the process of elimination. Eliminate option 2 first because oral fluids would not be given to a client who is difficult to arouse because of the risk of aspiration. From the remaining options, select option 4 recalling that insulin therapy is guided by blood glucose measurement. In addition, option 4 would provide additional data, which can then be reported to the physician. Review the immediate nursing interventions, if you had difficulty with this question, if hyperglycemia or hypoglycemia develops in a client. Level of Cognitive Ability: Application Client Needs: Physiological Integrity Integrated Process: Nursing Process/Implementation Content Area: Adult Health/Endocrine Reference: Linton, A., & Maebius, N. (2003). Introduction to medical-surgical nursing (3rd ed.). Philadelphia: W.B. Saunders, p. 921. 1693. A nurse is preparing for intershift report when a nurse’s aide pulls an emergency call light in a client’s room. Upon answering the light, the nurse finds a postoperative client experiencing tachycardia and tachypnea. The client’s blood pressure is 88/60 mm Hg. Which action should the nurse take first? 1. Check the hourly urine output 2. Check the intravenous (IV) site for infiltration 3. Place the client in modified Trendelenburg’s position 4. Check the client’s pulse oximetry Answer: 3 Rationale: The client is exhibiting signs of shock and requires emergency intervention. Placing the client in modified Trendelenburg’s position increases blood return from the legs, which increases venous return and subsequently the blood pressure. The nurse can then verify the client’s blood volume status by assessing the urine output and ensuring that the IV is infusing without complications. The nurse would also check the client’s pulse oximetry and notify the registered nurse. Test-Taking Strategy: Note the key word first. Use the ABCs—airway, breathing, circulation. Option 3 addresses the client’s circulatory status. Review the immediate nursing interventions when postoperative shock occurs if you had difficulty with this question. Level of Cognitive Ability: Application Client Needs: Physiological Integrity Integrated Process: Nursing Process/Implementation Content Area: Delegating/Prioritizing Reference: deWit, S. (2005). Fundamental concepts and skills for nursing. Philadelphia: W.B. Saunders, p. 750. 1694. A client with heart failure who is taking furosemide (Lasix) and digoxin (Lanoxin) calls the nurse and complains of anorexia and nausea. The nurse should take which action? 1. Hold the morning dose of furosemide 2. Check the result of the potassium level drawn 3 hours ago 3. Administer the daily dose of digoxin

PN~CD~Questions~1601-1700 -

57 -

4. Administer an antiemetic Answer: 2 Rationale: Anorexia and nausea are two of the common symptoms associated with digoxin toxicity, which is compounded by hypokalemia. The nurse should first check the results of the potassium level. This would provide additional data to report to the physician, which is a key follow-up nursing action. The nurse would not hold the furosemide without an order to do so, given the information provided. The nurse would withhold the digoxin and notify the registered nurse who would contact the physician since digoxin toxicity is suspected. The nurse would not administer an antiemetic without further investigating the client’s problem. Test-Taking Strategy: Focus on the data in the question. Use the steps of the nursing process to answer the question. Option 2 is the only option that addresses data collection. Review the initial nursing interventions when digoxin toxicity is suspected if you had difficulty with this question. Level of Cognitive Ability: Application Client Needs: Physiological Integrity Integrated Process: Nursing Process/Implementation Content Area: Pharmacology Reference: Hodgson, B., & Kizior, R. (2005). Saunders nursing drug handbook 2005. Philadelphia: W.B. Saunders, p. 326. 1695. A nurse is caring for a client undergoing peritoneal dialysis. The nurse checks the client and notes that the drainage from the outflow catheter is cloudy. The nurse notifies the registered nurse and plans to take which action? 1. Stop the peritoneal dialysis 2. Obtain a culture and sensitivity of the drainage 3. Temporarily institute hemodialysis 4. Add antibiotics to the next several dialysis bags Answer: 2 Rationale: When the drainage becomes cloudy, peritonitis is suspected. A culture and sensitivity is obtained, and broad spectrum antibiotics are added to the dialysis solution, pending culture and sensitivity results. The dialysis solution may also be heparinized to prevent catheter occlusion. Some clients must switch to hemodialysis if peritonitis is severe or reoccurring, but the nurse does not make this decision. The peritoneal dialysis is not stopped. Test-Taking Strategy: Use the process of elimination. Eliminate option 3 because a nurse would not make a decision to institute hemodialysis. Next, eliminate option 1 because stopping the peritoneal dialysis will not affect the consistency of the outflow. Recall that cloudy dialysate indicates infection and in this case peritonitis. Knowing that a culture should be obtained before instituting antibiotic therapy would guide you to the correct option. Review the unexpected outcomes and the associated nursing interventions related to peritoneal dialysis if you had difficulty with this question. Level of Cognitive Ability: Application Client Needs: Physiological Integrity Integrated Process: Nursing Process/Implementation Content Area: Adult Health/Renal

58

PN~CD~Questions~1601-1700 -

-

Reference: Christensen, B., & Kockrow, E. (2003). Adult health nursing (4th ed.). St. Louis: Mosby, p. 445. 1696. A nurse is assessing the chest tube drainage system of a postoperative client who had a right upper lobectomy. The closed drainage system has 300 mL of bloody drainage, and the nurse notes intermittent bubbling in the water seal chamber. One hour following the initial assessment, the nurse notes that the bubbling in the water seal chamber is now constant, and the client appears dyspneic. Based on these findings, the nurse should first check: 1. The client’s lung sounds 2. The client’s vital signs 3. The chest tube connections 4. The amount of drainage Answer: 3 Rationale: The client’s dyspnea is most likely related to an air leak caused by a loose connection. Other causes might be a tear or incision in the pulmonary pleura, which requires physician intervention. Although the other options are correct, they should be performed after initial attempts to locate and correct the air leak. It only takes a moment to check the connections, and if a leak is found and corrected, the client’s symptoms should resolve. The nurse would also notify the registered nurse. Test-Taking Strategy: Note the key word first and focus on the data in the question. Recalling that a constant bubbling in the water seal chamber could indicate a leak in the system will direct you to option 3. Review care to the client with a closed chest tube drainage system if you had difficulty with this question. Level of Cognitive Ability: Application Client Needs: Physiological Integrity Integrated Process: Nursing Process/ Implementation Content Area: Delegating/Prioritizing Reference: Christensen, B., & Kockrow, E. (2003). Adult health nursing (4th ed.). St. Louis: Mosby, p. 385. 1697. A nurse is monitoring a new mother for signs of postpartum depression. Which of the following, if noted in the new mother, would indicate the need for further data collection related to this form of depression? 1. The mother is caring for the infant in a loving manner 2. The mother constantly complains of tiredness and fatigue 3. The mother demonstrates an interest in the surroundings 4. The mother looks forward to visits from the father of the newborn Answer: 2 Rationale: Postpartum depression is not the normal depression that many new mothers experience from time to time. The woman experiencing depression shows less interest in her surroundings and a loss of her usual emotional response toward the family. The woman is also unable to show pleasure or love and may have intense feelings of unworthiness, guilt, and shame. The woman often expresses a sense of loss of self. Generalized fatigue, complaints of ill health, and difficulty in concentrating are also present. The mother would have little interest in food and experience sleep disturbances.

PN~CD~Questions~1601-1700 -

59 -

Test-Taking Strategy: Focus on the issue of the question. Note the key words need for further assessment. Use the process of elimination noting that options 1, 3, and 4 identify positive maternal behaviors. If you had difficulty with this question, review the clinical manifestations of postpartum depression. Level of Cognitive Ability: Analysis Client Needs: Psychosocial Integrity Integrated Process: Nursing Process/Data Collection Content Area: Maternity/Postpartum References: Lowdermilk, D., & Perry, S.E. (2004). Maternity & women’s health care (8th ed.). St. Louis: Mosby, p. 974. Murray, S., McKinney, E., & Gorrie, T. (2002). Foundations of maternal-newborn nursing (3rd ed.). Philadelphia: W.B. Saunders, p. 797. 1698. A client with pneumonia is admitted to the hospital, and the physician writes orders for the client. Which of the following orders written by the physician would the nurse complete first? 1. Obtain a culture and sensitivity of sputum 2. Administer prescribed antibiotic 3. Encourage the use of an incentive spirometry 4. Increase the intake of oral fluids Answer: 1 Rationale: A culture and sensitivity should be obtained before any antibiotic therapy is begun to prevent masking the microorganisms identified in the culture. Options 2, 3, and 4 are standard parts of therapy for pneumonia, but are not completed first. Test-Taking Strategy: Note the key word first. Remember that a culture and sensitivity specimen is always obtained before initiating antibiotic therapy. Review care to the client with pneumonia if you had difficulty with this question. Level of Cognitive Ability: Application Client Needs: Physiological Integrity Integrated Process: Nursing Process/Implementation Content Area: Delegating/Prioritizing Reference: Christensen, B., & Kockrow, E. (2003). Adult health nursing (4th ed.). St. Louis: Mosby, p. 381. 1699. A nurse is performing an assessment on a 2-day postpartum mother. The mother complains of severe pain and an intense feeling of swelling and pressure in the vulvar area. After hearing these complaints, the nurse specifically checks the client’s: 1. Episiotomy for drainage 2. Rectum for hemorrhoids 3. Vulva for a hematoma 4. Vagina for lacerations Answer: 3 Rationale: Hematoma is suspected when the client reports pain or pressure in the vulvar area. Massive hemorrhage can occur into the tissue, resulting in hypovolemia and shock; therefore the client’s complaints must be assessed so that interventions may begin immediately. The client’s complaints are not related to options 1, 2, or 4.

PN~CD~Questions~1601-1700 -

60 -

Test-Taking Strategy: Focus on the data in the question and note the key words specifically checks. Note the relation between the words “vulvar area” in the question and “vulva” in the correct option. Review the signs and symptoms of a vulvar hematoma and the associated nursing interventions if you had difficulty with this question. Level of Cognitive Ability: Analysis Client Needs: Physiological Integrity Integrated Process: Nursing Process/Implementation Content Area: Maternity/Postpartum Reference: Lowdermilk, D., & Perry, S.E. (2004). Maternity & women’s health care (8th ed.). St. Louis: Mosby, p. 1037. 1700. A nurse notes bilateral 2+ edema in the lower extremities of a client with known coronary artery disease who was admitted to the hospital 2 days ago. Based on this finding, the nurse implements which action? 1. Reviews the intake and output records for the last 2 days 2. Changes the time of diuretic administration from morning to evening 3. Requests a sodium restriction of 1 g/day from the physician 4. Orders daily weights starting on the following morning Answer: 1 Rationale: Edema is the accumulation of excess fluid in the interstitial spaces, which can be determined by intake greater than output and by a sudden increase in weight (2.2 lb = 1 kg). To determine the extent of fluid accumulation, the nurse first reviews the intake and output records for the last 2 days. Diuretics should be given in the morning whenever possible to prevent nocturia. Strict sodium restrictions are reserved for clients with severe symptoms. Test-Taking Strategy: Use the steps of the nursing process to answer the question. Option 1 is the only option that addresses data collection. Review nursing assessments with fluid imbalance if you had difficulty with this question. Level of Cognitive Ability: Application Client Needs: Physiological Integrity Integrated Process: Nursing Process/Implementation Content Area: Adult Health/Cardiovascular Reference: Black, J., & Hawks, J. (2005). Medical-surgical nursing: Clinical management for positive outcomes (7th ed.). Philadelphia: W.B. Saunders, pp. 216, 1567.

More Documents from "Linda Kuglarz"

Silvestri Chapter 21 Ed#551
October 2019 28
Silvestri101-200
October 2019 50
Silvestri1301-1400
October 2019 51
Silvestri Chapter 45 Ed#569
October 2019 37
Silvestri Chapter 32 Ed#55c
October 2019 41
Silvestri Chapter 07 Ed#543
October 2019 36